Você está na página 1de 138

Azyma142@hotmail.

com 0560903480

1- Which of the following anatomical terms best describes the motion in the picture
above:
a- Adduction
b- Abduction
c- Supination
d- Pronation

2- Which of the above locations represents the best sites to inspect chest retractions in a
child with lower respiratory tract disorder:
a- A&B
b- B&C
c- C&D
d- D&E
1
Azyma142@hotmail.com 0560903480

3- Your patient's finger stick blood sugar at 07:30 is 300 mg/dl, according to the
following sliding scale; how many unites you would give your patient:
a- 6 i.u
b- 8 i.u
c- 14 i.u
d- 18 i.u

4- The nurse is caring for a diabetic child whose 11 a.m. blood glucose monitoring
check is 269 mg/dL. The physician orders the following coverage schedule:

150 to 200 mg/dL2 units of Humulin R


201 to 250 mg/dL4 units of Humulin R
251 to 300 mg/dL6 units of Humulin R
301 to 350 mg/dL8 units of Humulin R
351 to 399 mg/dL10 units of Humulin R
Over 400 mg/dLcall the physician

According to the coverage schedule; how many unites you would give your patient:
a- 4 units
b- 6 units
c- 8 units
d- 10 units

2
Azyma142@hotmail.com 0560903480

5- The following syringe contains morphine sulphate in a concentration of 2 mg/cc, how


much total morphine this syringe contains:
a- 0.5 mg
b- 1 mg
c- 2 mg
d- 4 mg

6- The following syringe contains 2 gm of morphine sulphate, how many grams are
there in 1 ml:
a- 0.5 gm
b- 1 gm
c- 2 gm
d- 4 gm

7- The following condition may develop in females due to an increase in the production
of which of the following hormones:
a- Androgens
b- Progesterone
c- Estrogen
d- Growth

8- What is the name of The process of movement of a dissolved substance (solutes)


from an area of high concentration to an area of low concentration:
a- Effusion
b- Diffusion
c- Emulsion
d- Excretion
3
Azyma142@hotmail.com 0560903480

9- Which of the following is the incision site for an open cholecystectomy operation:
a- 1
b- 2
c- 3
d- 4

10- Which of the following is the incision site for an open gastrectomy operation:
a- 1
b- 2
c- 3
d- 4

11- Which of the following is the incision site for surgical appendectomy operation:
a- A
b- B
c- C
d- D

4
Azyma142@hotmail.com 0560903480

12- What is the most probable type of arrhythmia in the following ECG (lead ll):
a- Atrial fibrillation
b- Atrial flutter
c- Ventricular fibrillation
d- Supra-ventricular fibrillation

13- What is the most probable type of arrhythmia in the following ECG (lead V1):
a- Atrial fibrillation
b- Atrial flutter
c- Ventricular tachycardia
d- Supra-ventricular fibrillation

14- The following picture suggests that the patient has:


a- Renal disease
b- Hepatic disease
c- Pulmonary disease
d- Cerebral disease

15- A patient with colostomy, what type of stool you should suspect:
a- With a sigmoid colostomy, the feces are formed
b- With a descending colostomy, the feces are semiformed
c- With a transverse colostomy, the feces are unformed
d- With an ascending colostomy, the feces are fluid

5
Azyma142@hotmail.com 0560903480

16- When a patient is vomiting postoperatively, which of the following positions


would the nurse use to protect the patient?

a- c-

b- d-

17- The following picture represents the pain distribution of which nerve affection:
a- C2
b- C6
c- T3
d- T6

18- On assessing the clients pressure ulcer (see photo), the nurse would document
this as which stage:
a- Stage I
b- Stage II
c- Stage III
d- Stage IV

6
Azyma142@hotmail.com 0560903480

19- The following picture represents an auto immune disease known as:
a- Psoriasis
b- Systemic Lupus Erythematosus
c- Molluscum Contagiosum
d- Eczema

20- What is the name of the equipment this picture shows?


a- Oropharyngeal airway (OPA)
b- Nasopharyngeal airway (NPA)
c- Laryngeal mask airway (LMA)
d- Urinary catheter

21- A client is being prepared for a thoracentesis. A nurse assists the client to which
position for the procedure:
a- Lying in bed on the affected side
b- Lying in bed on the unaffected side
c- Sims position with the head of the bed flat
d- Prone with the head turned to the side and supported by a pillow

7
Azyma142@hotmail.com 0560903480

22- A nurse is preparing to insert a nasogastric tube into a client. The nurse places the
client in which position for insertion:
a- Right side c- High Fowlers
b- Low Fowlers d- Supine with the head flat

23- A nurse develops a plan of care for a client with deep vein thrombosis. Which client
position or activity in the plan will be included:
a- Out-of-bed activities as desired
b- Bedrest with the affected extremity kept flat
c- Bedrest with elevation of the affected extremity
d- Bedrest with the affected extremity in a dependent position

24- A client in labor is transported to the delivery room and is prepared for a cesarean
delivery. The client is transferred to the delivery room table, and the nurse places
the client in the:
a- Trendelenburg's position with the legs in stirrups
b- Semi-Fowler position with a pillow under the knees
c- Prone position with the legs separated and elevated
d- Supine position with a wedge under the right hip

25- The nurse is caring for a client who is 1 day postoperative for a total hip
replacement. Which is the best position in which the nurse should place the client:
a- Side-lying on the operative side
b- On the nonoperative side with the legs abducted
c- Side-lying with the affected leg internally rotated
d- Side-lying with the affected leg externally rotated

8
Azyma142@hotmail.com 0560903480

26- Which of the following is the best position to increase the brain perfusion( cerebral
perfusion):
a- Prone c- Semi- Fowlers
b- Supine d- Trendelenburgs

27- A nurse is administering a cleansing enema to a client with a fecal impaction.


Before administering the enema, the nurse places the client in which position:
a- Left Sims position
b- Right Sims position
c- On the left side of the body, with the head of the bed elevated 45 degrees
d- On the right side of the body, with the head of the bed elevated 45 degrees

28- A client has just returned to a nursing unit after an above-knee amputation of the
right leg. A nurse places the client in which position:
a- Prone
b- Reverse Trendelenburgs
c- Supine, with the amputated limb flat on the bed
d- Supine, with the amputated limb supported with pillows

29- A nurse is caring for a client with a severe burn who is scheduled for an autograft
to be placed on the lower extremity. The nurse develops a postoperative plan of care
for the client and includes which of the following in the plan:
a- Maintain the client in a prone position
b- Elevate and immobilize the grafted extremity
c- Maintain the surgical extremity in a flat position
d- Keep the surgical extremity covered with a blanket

9
Azyma142@hotmail.com 0560903480

30- A nurse is preparing to care for a client who has returned to the nursing unit
following cardiac catheterization performed through the femoral artery. The nurse
checks the physicians prescription and plans to allow which client position or
activity following the procedure:
a- Bedrest in high Fowlers position
b- Bedrest with bathroom privileges only
c- Bedrest with head elevation at 60 degrees
d- Bedrest with head elevation no greater than 30 degrees

31- After the client undergoes a total hip replacement, how should the nurse position
the affected hip?
a- Adduct the hip c- Flex the hip
b- Abduct the hip d- Extend the hip

32- A client has an open reduction and internal fixation for a fractured hip.
Postoperatively the nurse should position the client's affected extremity in:
a- External rotation
b- Slight hip flexion
c- Moderate abduction
d- Anatomical body alignment

33- Which of the following positions should the nurse place a client for rectal tube
insertion:
a- left lateral position with hyperextension of right knee
b- left lateral position with hyperflextion of left knee
c- left lateral position with hyperflextion of right knee
d- left lateral position with hyperextension of left knee

10
Azyma142@hotmail.com 0560903480

34- Which of the following position is appropriate for the patient to use for self-
administration of fleet enema:
a- Left lateral position right leg flex
b- Face down and leg flex
c- Face upward and leg free (on back and flex hip and knee)
d- Chest- knee position

35- The best position for a child with myelomeningocele is?


a- Prone
b- supine
c- semi-fowlers
d- modified trendelenburg

36- Unless the physician orders otherwise, in which position should the nurse place the
infant during the postoperative period of surgical repair of the myelomeningocele:
a- Supine
b- Prone
c- Right or left side-lying
d- Whichever position is most comfortable for the infant

37- The nurse is caring for a client with an above-the-knee amputation (AKA). To
prevent contractures, the nurse should:
a- Place the client in a prone position 1530 minutes twice a day
b- Keep the foot of the bed elevated on shock blocks
c- Place trochanter rolls on either side of the affected leg
d- Keep the clients leg elevated on two pillows

11
Azyma142@hotmail.com 0560903480

38- Postoperative nursing care of the infant following surgical repair of a cleft lip
would include:
a- Feeding the infant with a spoon to avoid sucking
b- Positioning the infant on the abdomen to facilitate drainage
c- Applying elbow restraints to protect the surgical area
d- Providing minimal stimulation to prevent injury to the incision

39- The client has returned to the nursing unit following a right below-the-knee
amputation. How should the nurse position the client?
a- Supine with head turned to the side
b- With shock blocks placed under the foot of the bed
c- Semi-sitting position with knees bent
d- Left lateral with pillows between the knees

40- What position should the nurse place the head of the bed in to obtain the most
accurate reading of jugular vein distention:
a- High fowlers
b- Raised 10 degrees
c- Raised 30 degrees
d- Supine position

41- Which of the following body positions is best for a patient with increased ICP:
a- Prone
b- Trendelenburg
c- HOB elevated 90 degrees and hips flexed
d- HOB elevated 20 degrees

12
Azyma142@hotmail.com 0560903480

42- Immediately after a percutaneous liver biopsy, the nurse should place the client in
which of the following positions:
a- Left- sided lying
b- Right- sided lying
c- Lithotomy
d- Trendelenburg

43- An adult has low back pain. Which position is likely to be most comfortable for the
client:
a- Prone
b- Supine
c- Side-lying with knees exed
d- Semi-sitting with legs extended

44- When planning the clients postoperative care, which is the least desirable position
in which the nurse can place the client:
a- Lying prone
b- Lying supine
c- Sitting in a chair
d- Standing to shower

45- Which of the following is the appropriate position for a patient who is suffering
from acute asthma attack is:
a- Right lateral
b- Supine
c- High fowler
d- Prone
13
Azyma142@hotmail.com 0560903480

46- A patient with COPD is admitted to the hospital. How can the nurse best position
the patient to improve gas exchange:
a- Sitting up at the bedside in a chair and leaning slightly forward
b- Resting in bed with the head elevated to 45 to 60 degrees
c- In the Trendelenburg's position with several pillows behind the head
d- Resting in bed in a high-Fowler's position with the knees flexed

47- Following a tympanoplasty, the nurse should maintain the client in which
position:
a- Semi-Fowlers with the operative ear facing down
b- Low Trendelenburgs with the head in neutral position
c- Flat with the head turned to the side with the operative ear facing up
d- Supine with a small neck roll to allow for drainage

48- Which of the following is the proper position for a patient undergoing a
cardiac surgery:
a- Supine
b- Prone
c- Lateral
d- Fowlers

49- A nurse is caring for a toddler after surgical repair of a cleft palate. The nurse
should position the child:
a- On his back
b- On his stomach
c- On his back with his head slightly elevated
d- For comfort
14
Azyma142@hotmail.com 0560903480

50- The nurse observes that the infant's anterior fontanelle is bulging after
placement of a ventriculoperitoneal shunt. The nurse positions this infant:
a- Prone, with the head of the bed elevated
b- Supine, with the head flat
c- Side-lying on the operative side
d- In a semi-fowler's position

51- A nurse is providing care for a client following surgery to remove a cataract
from the right eye. In which position should the nurse place the client?
a- Right-side lying c- Supine
b- Prone d- Trendelenburgs

52- An adult is admitted for a neurological workup and is scheduled for a spinal
tap. When preparing the client for the procedure, the nurse should position the
client in which position:
a- Prone
b- On the side with knees drawn up to chest
c- Lithotomy
d- Semi-sitting

53- When suctioning the oropharynx, which of the following is the proper position
of the patient:
a- Prone
b- Supine
c- Semi- Fowlers
d- Trendelenburgs

15
Azyma142@hotmail.com 0560903480

54- To prevent headache after spinal anesthesia the patient should be positioned:
a- Semi- fowlers
b- Flat on bed for 6 to 8 hours
c- Prone position
d- Modified Trendelenburg

55- If a patient develops autonomic hyperreflexia the first action his caring nurse
should take is to:
a- Elevate head of bed 90 degree (move from supine to sitting)
b- Make the bed in flat position
c- Apply ice on the axillary and groin
d- But the patient in trendelenburg position

56- To protect susceptible patients in the hospital from aspiration pneumonia, the
nurse will plan to:
a- Turn and reposition immobile patients at least every 2 hours.
b- Position patients with altered consciousness in lateral positions.
c- Monitor frequently for respiratory symptoms in patients who are immunosuppressed.
d- Provide for continuous subglottic aspiration in patients receiving enteral feedings.

57- Which nursing action is essential when providing continuous enteral feeding?
a- Elevating the head of the bed
b- Positioning the patient on the left side
c- Warming the formula before administrating it
d- Hanging a full days worth formula at one time

16
Azyma142@hotmail.com 0560903480

58- Which of the following positions is appropriate for the patient who has
hypovolemic shock:
a- Prone
b- Supine
c- Semi-fowlers
d- Flat with elevating the feet

59- When caring for a patient admitted post-stroke (CVA) who has altered
consciousness, the nurse should place the patient in which position:
a- Side-lying c- Prone
b- Supine d- Semi-fowler's

60- A nurse is providing instructions to a client and the family regarding home care
after right eye cataract removal. Which statement by the client would indicate an
understanding of the instructions:
a- I should not sleep on my left side.
b- I should not sleep on my right side.
c- I should not sleep with my head elevated.
d- I should not wear my glasses at any time.

61- The day after an amputation, the client begins to hemorrhage from his stump.
What action should the nurse take first:
a- Apply a pressure dressing to the stump
b- Place a tourniquet above the stump
c- Notify the physician
d- Apply an ice pack to the stump

17
Azyma142@hotmail.com 0560903480

62- A 72-year-old male client has a total hip replacement for long-standing
degenerative bone disease of the hip. When assessing this client postoperatively,
the nurse considers that the most common complication of hip surgery is:
a- Pneumonia c- Wound infection
b- Hemorrhage d- Pulmonary embolism

63- The nurse is caring for a client who had a right below-the-knee amputation
three days ago. The client complains of pain in the right foot and asks for pain
medication. What nursing action is appropriate initially?
a- Elevate the stump
b- Administer a placebo
c- Administer ordered medications
d- Encourage the client to discuss his feelings

64- During the assessment of an injury for a possible fracture, which of the
following manifestations would make you suspicious that a fracture has
occurred:
a- Impaired sensation c- Loss of function
b- Hotness d- Pointed tenderness

65- A client who has had an above-the-knee amputation develops a dime-sized


bright red spot on the dressing after 45 minutes in the postanesthesia recovery
unit. The nurse should:
a- Elevate the stump
b- Reinforce the dressing
c- Call the surgeon
d- Draw a mark around the site
18
Azyma142@hotmail.com 0560903480

66- How you will interfere to relieve swelling of the lower extremity in a cast for 3 days
now:
a- Change the cast c- Elevate the extremity
b- Call the doctor d- Give massage

67- The nurse is developing a bowel-retraining plan for a client with multiple sclerosis.
Which measure is likely to be least helpful to the client:
a- Limiting fluid intake to 1000mL per day
b- Providing a high-roughage diet
c- Elevating the toilet seat for easy access
d- Establishing a regular schedule for toileting

68- A diabetic patient with foot gangrene undergone above knee amputation, while the
nurse changing the dressing he complains of pain on the same knee which was
amputated. What should the nurse do:
a- Inform physician about it
b- Re-do dressing to Assessment the wound
c- Psychiatry consultation to the patient because he wound above knee
d- Give analgesic as needed

69- A diabetic patient with foot gangrene undergone above knee amputation he
complain of pain and swelling at the wound site which is oozing bus and has a bad
odor, the physician ordered cephalexin and metronidazole. Which of the following
you should do immediately:
a- Give cephalexin direct first action
b- Give metronidazole direct first action
c- Do wash on wound with N/Sand put bacitracin
d- Do wash on wound with N/Sand put hydrocortisone
19
Azyma142@hotmail.com 0560903480

70- The physician has prescribed a cleansing enema to a client scheduled for colon
surgery. The nurse would place the client:
a- Prone
b- Supine
c- Left sims ( left lateral )
d- Dorsal recumbent

71- At which side of the patient will the nurse stand when inserting a rectal enema:
a- Right side
b- Left side
c- Any side
d- Both sides

72- While undergoing a soapsuds enema, the client complains of abdominal cramping.
The nurse should:
a- Immediately stop the infusion
b- Lower the height of the enema container
c- Advance the enema tubing 2 to 3 inches
d- Clamp the tubing

73- The nurse must administer an enema to an adult patient with constipation. Which
of the following would be a safe and effective distance for the nurse to insert the
tubing into the patient's rectum:
a- 1:2 cm c- 5.5:6.5 cm
b- 3:4 cm d- 6.5:8 cm

20
Azyma142@hotmail.com 0560903480

74- The nurse is caring for a patient receiving IV furosemide (Lasix) 40 mg and
Enalapril (Vasotec) 5 mg PO bid for ADHF with severe orthopnea. When evaluating
the patient response to the medications, the best indicator that the treatment has
been effective is:
a- Weight loss of 2 pounds overnight.
b- Improvement in hourly urinary output.
c- Reduction in systolic bp.
d- Decreased dyspnea with the head of the bed at 30 degrees.

75- The nurse in preparing to insert RYLES tube (NGT) into an infant, the nurse
knows that the length of the tube should be taken as following:
a- From the nose down to the chin and then to the umbilicus
b- From the nose to the earlobe and then to the xiphoid process
c- From the nose to the mouth to the xiphoid process
d- From the nose to the earlobe to the umbilicus

76- The nurse is caring for a client who has had a chest tube inserted and connected to
water seal drainage. The nurse determines the drainage system is functioning
correctly when which of the following is observed:
a- Continuous bubbling in the water seal chamber
b- Fluctuation in the water seal chamber
c- Suction tubing attached to a wall unit
d- Vesicular breath sounds throughout the lung fields

21
Azyma142@hotmail.com 0560903480

77- The nurse is caring for a client who has just had a chest tube attached to a water
seal drainage system. To ensure that the system is functioning effectively the nurse
should:
a- Observe for intermittent bubbling in the water seal chamber
b- Flush the chest tubes with 30-60 ml of NSS every 4-6 hours
c- Maintain the client in an extreme lateral position
d- Strip the chest tubes in the direction of the client

78- The nurse enters the room of a client who has a chest tube attached to a water seal
drainage system and notices the chest tube is dislodged from the chest. The most
appropriate nursing intervention is to:
a- Notify the physician
b- Insert a new chest tube
c- Cover the insertion site with petroleum gauze
d- Instruct the client to breathe deeply until help arrives

79- which type of isolation category is indicated for patient with tuberculosis:
a- Airborne isolation c- Reverse isolation
b- Strict isolation d- Contact isolation

80- While attempting to get out of bed, a patient accidentally disconnects the chest tube
from the Pleur-evac drainage system. Which of the following actions should the
nurse take first?
a- Insert the end of the chest tube in a container of sterile solution
b- Clamp the chest tube near the Pleur-evac drainage system
c- Raise the end of the chest tube above the level of the insertion of the chest tube
d- Apply pressure dressing to the chest tube insertion site

22
Azyma142@hotmail.com 0560903480

81- which type of isolation category is indicated for a burn patient:


a- Airborne isolation
b- Strict isolation
c- Reverse isolation
d- No isolation required

82- which type of isolation category is indicated for patient with diphtheria:
a- Airborne c- Contact
b- Droplet d- Blood

83- A 68-years-old woman diagnosed with thrombocytopenia due to acute lymphocytic


leukemia is admitted to the hospital. The nurse should assign the patient to a:
a- To a private room so she will not infect other patients and health care workers.
b- To a private room so she will not be infected by other patients and health care
workers.
c- To a semiprivate room so she will have stimulation during her hospitalization.
d- To a semiprivate room so she will have the opportunity to express her feelings about
her illness.

84- A nurse who begins to administer medications to a client via a nasogastric feeding
tube suspects that the tube has become clogged. The nurse should take which safe
action first:
a- Aspirate the tube
b- Flush the tube with warm water
c- Prepare to remove and replace the tube
d- Flush with a carbonated liquid such as cola

23
Azyma142@hotmail.com 0560903480

85- Which of the following instructions is appropriate for the nurse to give to a female
client a who complains of abdominal upset after cholecystectomy operation:
a- Increase fluid intake
b- Avoid fatty meals
c- Increase protein intake
d- Daily exercise

86- Which of the following pulses should be checked before administrating Digoxin:
a- Apical pulse
b- Radial pulse
c- Femoral pulse
d- Dorsalis pedis pulse

87- which of the following interventions must the nurse take when administrating
digoxin to the patient:
a- Give him the medication with a glass of orange juice
b- Check him for signs of hypokalemia before giving the medication
c- Instruct him to place the medication under the tongue
d- Withhold the medication if his pulse is less than 60 beats/ minute

88- The nurse must withhold Digoxin from a patient if his pulse rate is:
a- Less than 45/m
b- More than 60/m
c- Less than 60/m
d- More than 100/m

24
Azyma142@hotmail.com 0560903480

89- The nurse is assessing the client for possible evidence of digitalis toxicity. The nurse
acknowledges that which is included in the signs and symptoms for digitalis toxicity:
a- Pulse (heart) rate of 100 beats/min
b- Pulse of 72 with an irregular rate
c- Pulse greater than 60 beats/min and irregular rate
d- Pulse below 60 beats/min and irregular rate

90- A newly admitted client takes digoxin 0.25 mg/day. The nurse knows that the
serum therapeutic range for digoxin is:
a- 0.1 to 1.5 ng/mL
b- 0.5 to 2.0 ng/mL
c- 1.0 to 2.5 ng/mL
d- 2.0 to 4.0 ng/mL

91- A patient complains of severe pain which he stated to be 9/10, the physician
ordered morphine 50 mg IV every 4 hours, the last dose was given 2 hours ago, what
is the best action his caring nurse would take:
a- Give another dose of morphine
b- Inform the doctor to change the order
c- Distract the patient by TV, radio or games for 2 hours
d- Ignore the patient completely

92- The nurse is caring for a patient admitted 1 week ago with an acute spinal cord
injury. Which of the following assessment findings would alert the nurse to the
presence of autonomic dysreflexia (hyperreflexia)?
a- Tachycardia c- Hot, dry skin
b- Hypotension d- Throbbing headache

25
Azyma142@hotmail.com 0560903480

93- Which of the following is the best way for long term feeding for a patient
experiencing severe dysphagia:
a- NGT
b- Naso-dudenal tube
c- Gastrostomy
d- Parenteral

94- Patient records are as following, he had 650cc of IV fluid (n/s), 50 ml of orange
Juice in the NG tube, 100 water with the medications, and urinate 500 cc, and 100 cc
collected in the NG bag. What is the total intake for this patient?
a- 800 cc c- 200 cc
b- 650 cc d- 600 cc

95- The physician ordered to give the patient 10,000 unit of heparin, the preparation
available is 40,000/ml. how much would the nurse give this patient:
a- 4 ml c- 0.25 ml
b- 2.5 ml d- 25 ml

96- The nurse is caring for a client with a fracture. The client develops a deep vein
thrombosis in the opposite extremity. Physician orders include a heparin drip of
D5W 250 mL with heparin 12,500 units at 16 mL/hr. How many units of heparin is
this client receiving per hour:
a- 200 unit/h
b- 400 unit/h
c- 600 unit/h
d- 800 unit/h

26
Azyma142@hotmail.com 0560903480

97- A patient suffers from diarrhea and muscle weakness, the nurse would assess this
patient for which of the following abnormal laboratory blood tests:
a- Blood calcium c- Blood potassium
b- Blood magnesium d- Blood sodium

100- When assessing for therapeutic effects of mannitol, the nurse would expect to see:
a- Decreased intracranial pressure
b- Decreased excretion of therapeutic medications
c- Increased urine osmolality
d- Decreased serum osmolality

101- Which drug would be used to treat a patient who has increased intracranial
pressure (ICP) resulting from head trauma after an accident:
a- Mannitol
b- Atropine sulfate
c- Epinephrine hydrochloride
d- Sodium bicarbonate

102- Which of the following is the drug of choice for treatment of cerebral edema:
a- Mannitol
b- Atropine sulfate
c- Epinephrine hydrochloride
d- Sodium bicarbonate

27
Azyma142@hotmail.com 0560903480

103- The nurse would assess which laboratory value to determine the effectiveness of
intravenous heparin:
a- Complete blood count (CBC)
b- Activated partial thromboplastin time (aPTT)
c- Prothrombin time (PT)
d- BUN

104- The antidote to heparin is:


a- Aspirin c- Warfarin
b- Vitamin k d- Protamine sulfate

105- How do we monitor the effectiveness of Coumadin (Warfarin):


a- Monitor vital signs
b- PT/INR
c- Weight
d- Ask the patient how they feel

106- Which medication is used to treat a patient suffering from severe adverse effects of
a narcotic analgesic:
a- Naloxone (Narcan)
b- Acetylcysteine (Mucomyst)
c- Methylprednisolone (Solu-Medrol)
d- Protamine sulfate

107- A patient who has been anticoagulated with warfarin (Coumadin) is admitted with
gastrointestinal bleeding. The nurse will anticipate administering which substance:
a- Vitamin E c- Protamine sulfate
b- Vitamin K d- Calcium gluconate

28
Azyma142@hotmail.com 0560903480

108- While admitting a patient for treatment of an acetaminophen overdose, the nurse
prepares to administer which of the following medications to prevent toxicity:
a- Naloxone (Narcan)
b- Acetylcysteine (Mucomyst)
c- Methylprednisolone (Solu-Medrol)
d- Vitamin K

109- If the nurse gave a patient antihypertensive drug that is prescribed for another
patient, what should she do:
a- Ignore what she has done
b- Call urgent CPR
c- Document that in nursing report
d- Inform doctor after 2 days

110- When a physician orders an arterial blood gas, which artery is the appropriate one
to use to obtain the specimen:
a- Cerebral artery
b- Radial artery
c- Carotid artery
d- Ulnar artery

111- A client has an order to have a set of arterial blood gases (ABG's) drawn. The
intended site is the radial artery. The nurse ensures that which of the following is
positive before the ABGs are drawn
a- Homans sign c- Babinski reflex
b- Brudzinski's sign d- Allen's test

29
Azyma142@hotmail.com 0560903480

112- A client is scheduled for blood to be drawn from the radial artery for an ABG
determination. A nurse assists with performing Allen's test before drawing the
blood to determine the adequacy of the:
a- Ulnar circulation
b- Carotid circulation
c- Femoral circulation
d- Brachial circulation

113- A nurse has administered approximately half of an enema solution to a


preoperative client when the client complains of pain and cramping. Which
nursing action is the most appropriate:
a- Raise the enema bag so that the solution can be instilled quickly.
b- Clamp the tubing for 30 seconds and restart the flow at a slower rate.
c- Reassure the client and continue the flow.
d- Discontinue the enema and notify the physician

114- When caring for a client who is receiving total parenteral nutrition (TPN), which
of the following complications would be most important for the nurse to assess:
a- Chest pain
b- Hemorrhage and air embolus
c- Pneumonia and hyperglycemia
d- Electrolyte imbalance and sepsis

115- A client has a nasogastric tube after a gastric resection. The nurse should expect to
observe:
a- Vomiting
b- Gastric distention
c- Intermittent periods of diarrhea
d- Bloody drainage for the first 12 hours
30
Azyma142@hotmail.com 0560903480

116- While assessing a client with hypoparathyroidism, the nurse taps the client's facial
nerve and observes twitching of the mouth and tightening of the jaw. The nurse
would document this finding as which of the following:
a- Positive Trousseau's sign
b- Positive Chvostek's sign
c- Tetany
d- Hyperactive deep tendon reflex

117- Which of the following clinical manifestations of type 2 diabetes occurs if glucose
levels are very high:
a- Hyperactivity
b- Blurred vision
c- Oliguria
d- Increased energy

118- Which of the following would the nurse expect to find in a client with severe
hyperthyroidism:
a- Tetany
b- Buffalo hump
c- Exophthalmos
d- Striae

119- A client is receiving long-term treatment with high-dose corticosteroids. Which of


the following would the nurse expect the client to exhibit:
a- Weight loss
b- Pale thick skin
c- Hypotension
d- Moon face
31
Azyma142@hotmail.com 0560903480

120- After undergoing a thyroidectomy, a client develops hypocalcemia and tetany.


Which electrolyte should the nurse anticipate administering:
a- Sodium phosphorus
b- Sodium bicarbonate
c- Calcium gluconate
d- Potassium chloride

121- The nurse who elicits a positive Chvosteks sign would suspect that the patient has
which condition:
a- Hyperkalemia
b- Hypocalcemia
c- Hypercalcemia
d- Hypernatremia

122- Which of the following is the best method to decrease confusion and irritability
for asthmatic patient:
a- Give antibiotic
b- Give sedative
c- Give vasodilator
d- Give oxygen

123- Which one of the baseline vital signs that has the most effect on spO2:
a- Heart rate c- Blood pressure
b- Respiratory rate d- Temperature

32
Azyma142@hotmail.com 0560903480

124- A patient is admitted to the emergency department with a stab wound to the right
chest. Air can be heard entering his chest with each inspiration. To decrease the
possibility of a tension pneumothorax in the patient, the nurse should:
a- Position the patient so that the right chest is dependent.
b- Administer high-flow oxygen using a non-rebreathing mask.
c- Cover the sucking chest wound with an occlusive dressing.
d- Tape a nonporous dressing on three sides over the chest wound.

125- A client with pneumonia has a temperature of 102.6 F (39.2 C), is


diaphoretic, and has a productive cough. The nurse should include which of
the following measures in the plan of care:
a- Position changes every 4 hours.
b- Nasotracheal suctioning to clear secretions.
c- Frequent linen changes
d- Frequent offering of a bedpan

126- The nurse reviews an arterial blood gas report for a client with chronic
obstructive pulmonary disease (COPD). PH 7.35; PC02 62; PO2 70; HCO3 34
the nurse should:
a- Apply a 100% non-rebreather mask.
b- Assess the vital signs.
c- Reposition the client.
d- Prepare for intubation

127- If the nurse notes the following symptoms after the client begins taking
sertraline (Zoloft), which one is most likely drug-related:
a- Polyuria c- Drooling
b- Diplopia d- Insomnia
33
Azyma142@hotmail.com 0560903480

128- Which of the following diets would be most appropriate for a client with
chronic obstructive pulmonary disease (COPD):
a- Low-fat, low-cholesterol diet.
b- Bland, soft diet.
c- Low-sodium diet.
d- High-calorie, high-protein diet.

129- The physician has prescribed sertraline (Zoloft) 50 mg daily for a client with
depression. Which finding should be reported to the physician?
a- The client takes Tagamet (cimetidine) for acid reflux
b- The client takes the medication with meals
c- The client takes the medication once a day at bedtime
d- The client takes Aleve (naproxen) for arthritis

130- A patient was diagnosed with depression 6 weeks ago, and was prescribed
anti-depressant drug.1 week ago the patient started to complain of irritability
and had difficult sleep for only 3 hours daily. Which of the following is the
best nursing action:
a- Inform the doctor to increase the dose
b- Ignore the patient complaint
c- Thats a side effect of the medication
d- Give the patient hypnotics

131- 16 years old girl tried to suicide by taking a large amount of valium
(benzodiazepines), the nurse knows that the antidote for this drug is:
a- Flumazenil c- Meperidine
b- Benztropine d- Naloxone

34
Azyma142@hotmail.com 0560903480

132- The nurse would expect to find which information when reviewing the history
of a client diagnosed with multiple sclerosis:
a- Visual problems
b- Increased sensitivity to pain
c- Ascending weakness and numbness
d- Confusion and disorientation

133- The antidote for morphine overdose is:


a- Naloxone (Narcan)
b- Flumazenil (Romazicon)
c- Benztropine (Cogentin)
d- Meperidine (Demerol)

134- The nurse assesses a respiratory rate of 10 on a client with cancer who has just
received a hydromorphone hydrochloride (Dilaudid) injection. Which drug
should the nurse be prepared to administer:
a- Naloxone (Narcan)
b- Flumazenil (Romazicon)
c- Benztropine (Cogentin)
d- Meperidine (Demerol)

135- Which of the following drugs is the antidote for magnesium toxicity:
a- Calcium gluconate (Kalcinate)
b- Hydralazine (Apresoline)
c- Naloxone (Narcan)
d- Rho (D) immune globulin (RhoGAM)

35
Azyma142@hotmail.com 0560903480

136- A client with myasthenia gravis is suspected of having cholinergic crisis.


Which of the following indicate that this crisis exists:
a- Ataxia c- Hypotension
b- Mouth sores d- Hypertension

137- A client with myasthenia gravis is receiving pyridostigmine (Mestinon). The


nurse monitors for signs and symptoms of cholinergic crisis caused by
overdose of the medication. The nurse checks the medication supply to ensure
that which medication is available for administration if a cholinergic crisis
occurs:
a- Vitamin K
b- Atropine sulfate
c- Protamine sulfate
d- Acetylcysteine (Mucomyst)

138- Which of the following is the purpose of providing atropine prior to anesthesia
and surgery:
a- To decrease salivation (decrease secretions)
b- To increase blood pressure
c- To facilitate breathing
d- To decrease muscle tone

139- A client is diagnosed as having secondary Cushings syndrome. The nurse


knows that the client has most likely been taking which medication:
a- Estrogen
b- Penicillin
c- Lovastatin
d- Prednisone
36
Azyma142@hotmail.com 0560903480

140- A patient has the following preoperative medication order: morphine 10 mg


with atropine 0.4 mg IM. The nurse informs the patient that this injection will:
a- Decrease nausea and vomiting during and after surgery
b- Decrease oral and respiratory secretions, thereby drying the mouth
c- Decrease anxiety and produce amnesia of the preoperative period
d- Induce sleep, so the patient will not be aware during transport to the operating
room

141- A 40-year-old woman is admitted in labor with high blood pressure, edema,
and proteinuria. She is started on magnesium sulfate. The nurse caring for her
should be sure to keep which drug at the bedside:
a- Calcium gluconate
b- Naloxone (Narcan)
c- Phenytoin (Dilantin)
d- Glucose

142- Which of the following diets would be most appropriate for a patient has
Cushing syndrome:
a- High protein diet
b- High potassium diet
c- High calcium diet
d- High iron diet

143- Vitamin K is prescribed for a neonate. A nurse prepares to administer the


medication in which muscle site:
a- Deltoid c- Vastus lateralis
b- Triceps d- Biceps

37
Azyma142@hotmail.com 0560903480

144- Which of the following interventions should be taken to help an older client to
prevent osteoporosis:
a- Decrease dietary calcium intake
b- Increase sedentary lifestyles
c- Increase dietary protein intake
d- Encourage regular exercise

145- An adult is admitted with heart failure. The nurse notes that he has neck vein
distention and slight peripheral edema. The nurse knows that these signs
indicate which of the following:
a- Pneumothorax
b- Right-sided heart failure
c- Cardiogenic shock
d- Left-sided heart failure

146- An adult client is admitted with a diagnosis of left-sided congestive heart


failure. Which assessment finding would most likely be present?
a- Distended neck veins
b- Dyspnea
c- Hepatomegaly
d- Pitting edema

147- When the nurse on duty accidentally bumps the bassinet, the neonate throws
out its arms, hands opened, and begins to cry. The nurse interprets this
reaction as indicative of which of the following reflexes:
a- Moro Reflex c- Grasping Reflex
b- Babinski reflex d- Tonic Neck Reflex
38
Azyma142@hotmail.com 0560903480

148- A patient who has required prolonged mechanical ventilation has the
following arterial blood gas results: pH 7.48, PaO2 85 mm Hg, PaCO2 32 mm
Hg, and HCO3 25 mEq/L. The nurse interprets these results as:
a- Metabolic acidosis
b- Metabolic alkalosis
c- Respiratory acidosis
d- Respiratory alkalosis

149- A patient has the following ABG results: pH 7.32, PaO2 88 mm Hg, PaCO2 37
mm Hg, and HCO3 16 mEq/L. The nurse interprets these results as:
a- Respiratory acidosis
b- Respiratory alkalosis
c- Metabolic acidosis
d- Metabolic alkalosis

150- The nurse forgot to give the patient his prescribed drug, when the head nurse
asked who is responsible for this the nurse replayed that was my wrong this
considered:
a- Responsibility c- Abandonment
b- Ignorance d- Honor

151- When the patient vomits postoperatively, the most important nursing
objective is to prevent:
a- Dehydration
b- Aspiration
c- Rupture of suture line
d- Met. Alkalosis
39
Azyma142@hotmail.com 0560903480

152- The nurse is discussing dietary sources of iron with a client who has iron
deficiency anemia. Which menu, if selected by the client, indicates the best
understanding of the diet?
a- Milkshake, hot dog, and beets
b- Beef steak, spinach, and grape juice
c- Chicken salad, green peas, and coffee
d- Macaroni and cheese, coleslaw, and lemonade

153- A client is recovering from abdominal surgery and has a large abdominal
wound. A nurse encourages the client to eat which food item that is naturally
high in vitamin C to promote wound healing:
a- Milk
b- Oranges
c- Bananas
d- Chickens

154- Which of the following reflexes is considered a normal reflex of the newborn:
a- Moro reflex
b- Rooting reflex
c- Planter reflex
d- Tonic neck reflex

155- Patient with aortic aneurysm, which of the following is the most dangerous
complications:
a- Embolism
b- Rupture
c- Stenosis
d- Hypotension

40
Azyma142@hotmail.com 0560903480

156- 56 years old patient suffers from heart failure, when the nurse examines his
lungs she noticed the presence of crackles which means that the patient has:
a- Right-sided- failure
b- Left-sided- failure
c- Biventricular failure
d- Congestive heart failure

157- The nurse teaches a client with chronic obstructive pulmonary disease
(COPD) to assess for signs and symptoms of right-sided heart failure. Which
of the following signs and symptoms should be included in the teaching plan:
a- Clubbing of nail beds
b- Hypertension
c- Peripheral edema
d- Increased appetite

158- A surgery procedure that must be done within 24 hours is called:


a- Selective surgery
b- Elective surgery
c- Urgent surgery
d- Emergency surgery

159- Early sign of ARDS in a patient at risk:


a- Elevated CO2 level
b- Hypoxia not responsive to O2 (cyanosis)
c- Metabolic acidosis
d- Severe, unexplained light imbalance

41
Azyma142@hotmail.com 0560903480

160- A nurse is assessing a female client with multiple trauma who is at risk for
developing acute respiratory distress syndrome. The nurse assesses for which
earliest sign of acute respiratory distress syndrome:
a- Bilateral wheezing
b- Inspiratory crackles
c- Intercostal retractions
d- Increased respiratory rate

161- 36 years old male patient complaining of fever and headache for 3 days now,
when examining this patent which of the following signs would indicate that
this patient may has meningitis:
a- Positive Kernigs sign
b- Negative Brudzinskis sign
c- Positive homans sign
d- Negative Kernigs sign

162- A patient presented with high fever, headache, vomiting and neck stiffness for
the past 3 days, which of the following is the first diagnostic intervention for
this patient:
a- Urine and stool analysis
b- Lumber puncture with CSF aspiration
c- Complete blood count
d- Chest and abdomen x-ray

163- MRSA (methicillin resistant Staphylococcus aureus) is transmitted by:


a- Contact c- Droplet
b- Airborne d- Standard

42
Azyma142@hotmail.com 0560903480

164- What is the best indicator of effective treatment of CHF in patient taking
Lasix?
a- Calculating total intake daily
b- Weighting himself daily
c- Calculating urine output daily
d- Assessing fluid status daily

165- What supplemental medication is most frequently ordered in conjunction with


furosemide (Lasix)?
a- Chloride c- Potassium
b- Digoxin d- Sodium

166- What is the first intervention for a client experiencing myocardial infarction?
a- Administer morphine
b- Administer oxygen
c- Administer sublingual nitroglycerine
d- Obtain an electrocardiogram

167- Which type of medications is commonly used to treat rheumatoid arthritis?


a- Glucocorticoids
b- Non- steroidal ant- inflammatory drugs (NSAIDs)
c- Antimalarial drugs
d- Gold salt

43
Azyma142@hotmail.com 0560903480

168- A male client undergoes total gastrectomy. Several hours after surgery, the
nurse notes that the clients nasogastric (NG) tube has stopped draining. How
should the nurse respond?
a- Notify the physician
b- Reposition the tube
c- Irrigate the tube
d- Increase the suction level

169- Postoperatively, patient is expected to void after:


a- 6-8 hours
b- 2-4 hours
c- 12-24 hours
d- 10-12 hours

170- A female client has severe menstrual pain, which of the following drugs you
would recommend for this patient to relieve her dysmenorrhea:
a- Ibuprofen c- Aspirin
b- Zantac d- Cortisone

171- The nurse must suction a child with a tracheostomy. Interventions should
include:
a- Encouraging the child to cough to raise the secretions before suctioning
b- Selecting a catheter with a diameter three fourths as large as the diameter of the
tracheostomy tube
c- Ensuring that each pass of the suction catheter take no longer than 5 seconds
d- Allowing the child to rest after every five times the suction catheter is passed

44
Azyma142@hotmail.com 0560903480

172- A 40 years old female complains of abdominal pain. It is worse after eating,
especially if she has a meal that is spicy or high in fat. She has tried over-the-
counter antacids, but they have not helped the pain. After examining her
abdomen, you strongly suspect cholecystitis. Which sign on examination
increases your suspicion for this diagnosis:
a- Psoas sign
b- Rovsings sign
c- Murphys sign
d- Grey turners sign

173- Which pulse should the nurse palpate during rapid assessment of an
unconscious male adult?
a- Radial c- Femoral
b- Brachial d- Carotid

174- The most important nursing measure in the prevention of thrombophlebitis


for the post-partum mother is:
a- Elastic stocking
b- Early ambulation
c- Anticoagulants
d- Isometric exercises

175- Which of the following is the best aid to prevent breast cancer:
a- Teaching women about breast cancer
b- Public knowledge about chemotherapy
c- To eat fruits and vegetables only
d- Encourage women to perform self-breast examination monthly

45
Azyma142@hotmail.com 0560903480

176- Which of the following factors would contribute to a high risk pregnancy:
a- Blood type O positive
b- First pregnancy at 33 years old
c- History of allergy to honey bee pollen
d- History of insulin dependent DM

177- When the nurse puts an infant with high temperature in cold water in order to
lower his temperature, she knows that the mechanism of heat loss is called:
a- Evaporation
b- Conduction
c- Radiation
d- Convection

178- A child is admitted to the hospital with a diagnosis of Wilm's tumor, Stage II.
Which of the following statements most accurately describes this stage:
a- The tumor is less than 3 cm. in size and requires no chemotherapy
b- The tumor did not extend beyond the kidney and was completely resected
c- The tumor extended beyond the kidney but was completely resected
d- The tumor has spread into the abdominal cavity and cannot be resected

179- When assessing a 12 year old child with Wilm's tumor, the nurse should keep
in mind that it most important to avoid which of the following:
a- Measuring the child's chest circumference
b- Palpating the child's abdomen
c- Placing the child in an upright position
d- Measuring the child's occipitofrontal circumference

46
Azyma142@hotmail.com 0560903480
180- The nurse is teaching a 45 year old woman how to increase the potassium in
her diet. The woman says she knows bananas are high in potassium, but she
doesn't like their taste. What foods should the nurse recommend the client
include in her diet:
a- Carrots, broccoli, yogurt
b- Rhubarb, tofu, celery
c- Potatoes, spinach, raisins
d- Onions, corn, oatmeal

181- When developing a plan of care for a patient with SIADH (syndrome of
inappropriate antidiuretic hormone secretion), which interventions will the
nurse include:
a- Encourage fluids to 2000 ml/day
b- Long-term fluid restriction
c- Monitor for increased peripheral edema
d- Keep head of bed elevated to 30 degrees

182- A client is admitted for treatment of the syndrome of inappropriate


antidiuretic hormone (SIADH). Which nursing intervention is appropriate?
a- Infusing I.V. fluids rapidly as ordered
b- Administering glucose-containing I.V. fluids as ordered
c- Restricting fluids
d- Encouraging increased oral intake

183- How streptococci infection is transmitted from person to another:


a- Airborne c- Slandered
b- Contact d- Blood

47
Azyma142@hotmail.com 0560903480

184- A hospitalized client is found to be comatose and hypoglycemic with a blood


sugar of 50 mg/dL. Which of the following would the nurse do first?
a- Infuse 1000 mL D5W over a 12-hour period
b- Encourage the client to drink orange juice with added sugar
c- Check the client's urine for the presence of sugar and acetone
d- Administer 50% glucose intravenously

185- A patient receives TPN, which of the following nursing actions is important to
minimize the risk for fluid volume excess:
a- Increase diuretic dose if swelling occurs
b- Limit the amount of free water in relation to sodium intake
c- Monitor his or her skin turgor
d- Weigh the patient daily on the same scale

186- Which of the following findings would most likely indicate the presence of a
respiratory infection in a client with asthma:
a- Cough productive of yellow sputum
b- Bilateral expiratory wheezing
c- Chest tightness
d- Respiratory rate of 30 breaths/ minute

187- When teaching a mother how to administer eye drops, where should the nurse
tell her to place them?
a- In the conjunctival sac that is formed when the lower lid is pulled down
b- Carefully under the eye lid while it is gently pulled upward
c- On the sclera while the child looks to the side
d- Anywhere as long as drops contact the eye's surface

48
Azyma142@hotmail.com 0560903480

188- One nursing intervention for patient with asthma is to facilitate removal of
secretions. This can be done by:
a- Encourage the patient to perform slow and shallow breathing
b- Encourage the patient to increase fluid intake
c- Encourage the patient to hyperventilate
d- Encourage the patient to decrease fluid intake

189- The nurse is caring for a 6-year-old boy following revision of


ventriculoperitoneal shunt. An expected nursing intervention is:
a- Request for an x-ray to evaluate shunt placement
b- Daily measurement of head circumference
c- Frequent palpation of the fontanels
d- Maintaining the child in a prone position

190- the patient have an itchy, erythematous, vesicular, weeping, and crusting
patches on his skin which skin disease is this considered:
a- Psoriasis
b- Impetigo
c- Eczema
d- Urticaria

191- Giving an intramuscular injection to an infant in the Gluteus Maximus muscle


may produce an injury to which of the following nerves:
a- Femoral nerve
b- Sciatic nerve
c- Vagus nerve
d- Popliteal nerve

49
Azyma142@hotmail.com 0560903480

192- Which of the following is one of the side effects of albuterol nebulizer:
a- Hypertension
b- Hypotension
c- Fast irregular heart beats
d- Tachypnea

193- A patient, age 49, returns from the postanesthesia care unit after a total
abdominal hysterectomy and bilateral salpingo-oophorectomy to treat cervical
cancer. Which nursing intervention has the highest priority at this time?
a- Monitor the patient for indications of hemorrhage
b- Assess the patients pain level and response to analgesics
c- Encourage the patient to do deep breathing and leg exercises
d- Provide emotional support to the patient

194- When planning the care of a client who has undergone an abdominal
hysterectomy, which nursing measure is most helpful for preventing
postoperative complications and facilitating an early discharge?
a- Reestablishing oral fluids and nutrition
b- Promoting ambulation and movement
c- Maintaining accurate intake and output
d- Exploring feelings about altered image

195- Which of the following is considered one of the common complications


following abdominal total hysterectomy:
a- DVT
b- Paralytic ileus
c- Constipation
d- Perforation

50
Azyma142@hotmail.com 0560903480

196- A client had a hysterectomy 10 hours ago. The nurse assesses the client and
finds that her blood pressure has fallen abruptly. Which action by the nurse is
most appropriate at this time?
a- Continue to monitor the blood pressure every 15 minutes
b- Document the information on the clients chart
c- Inform the surgeon about the clients condition
d- Change the client to a Fowlers position

197- The nurse is teaching a client regarding risk factors for stroke (CVA). The
greatest risk factor is which of the following:
a- Diabetes
b- Heart disease
c- Renal insufficiency
d- Hypertension

198- The nurse recognizes that the most common type of brain attack (CVA) is
related to which of the following:
a- Ischemia c- Headache
b- Hemorrhage d- Vomiting

199- A woman comes to the clinic and states, "I've been sick for so long! My eyes
have gotten so puffy, and my eyebrows and hair have become coarse and dry."
The nurse will assess for other signs and symptoms of:
a- Cachexia
b- Parkinson's syndrome
c- Myxedema
d- Scleroderma

51
Azyma142@hotmail.com 0560903480

200- A 32 year-old man is brought to the emergency department by a friend with


whom he had been playing football with earlier that evening. His symptoms
came on after twenty minutes of the football match. He appears anxious and
restless. Auscultation of the lung reveals wheezing on exhalation. The patient
states that he is unable to get a full breath of air. He had this problem as a
child a couple of times after exercising in cold weather.
Blood pressure 126/72mmhg
Heart rate 90/min
Respiratory rate 28/min shallow
Temperature 37.0C tympanic
Oxygen saturation 94% on room air
Which position is the best for this patient?
a- Supine
b- Reverse Trendelenburgs
c- Prone
d- High fowlers

201- A 43 years- old man is 30- hour post- operative following placement of a
partial thickness skin autograft for a burn injury on the lower anterior leg.
During a routine assessment, the nurse observes the wound is bleeding
continuously.
What is the priority nursing action?
a- Use a pen to outline and monitor the area
b- Perform a wound swab for laboratory analysis
c- Incise and drain fluids from the wound bed
d- Apply firm and direct pressure for 10 minutes

52
Azyma142@hotmail.com 0560903480

202- Coarse hair may indicate:


a- Dehydration c- Under nutrition
b- Inflammation d- Anemia

203- The nurse performs a home visit for a 32 years- old woman who had given
birth to her first infant three days before. The mother has concerns about
breastfeeding and the nurse observes the infant feeding. The mother sits
supported upright with cushions and the infant positioned in a cradle hold.
The infants head and body are aligned against the mothers abdomen. The
infant sucks intermittently with the lips turned outwards and the nipple in the
mouth.
Which intervention is most appropriate?
a- Reposition the infant
b- Assist the infant to turn lips inwards
c- Ensure the latch includes the areola
d- Stimulate the infant to suck constantly

204- A 52 year- old woman is scheduled to undergo an abdomino- perineal


resection in three days for removal of a cancer of the rectum. The nurse
reviews the care plan with the patient. The patient will receive prophylactic
antibiotics and will be given a mechanical bowel preparation the day before.
Which additional preparation should the patient undertake at this time?
a- Wear pressure stockings
b- Perform leg strengthening exercises
c- Maintain high- protein, low- residue diet
d- Take daily ferrous iron tablets

53
Azyma142@hotmail.com 0560903480

205- A patient is preparing for a scheduled hip replacement.


Which lab value should be reported to the physician?
Test Result Normal values
Sodium 145 134-164 mmol/L
Potassium 2.9 3.5-5.1 mmol/L
Calcium 2.80 2.15-2.62mmol/L
Magnesium 4.8 1.2-2 mmol/L
a- Sodium
b- Calcium
c- Potassium
d- Magnesium

206- A nurse is admitting a six month- old infant with pneumonia. Which of the
following interventions supports this infants emotional needs?
a- Allow the parents to leave the room during painful procedures
b- Encourage parents to distract the infant from crying
c- Interview the patents to learn the infants comforting habits
d- Enforce strict visiting schedule and routines

207- While caring for a patient in the post- anesthesia care unit (PACU) who has
developed hypovolemic shock, a nurse should position the patient:
a- Flat with legs elevated
b- In trendelenburg position
c- With the head of the bed elevated 45 degrees
d- Completely flat

54
Azyma142@hotmail.com 0560903480

208- A five years old child girl had presented with tenderness, headache and
petechiae. She was pale and complains of joint pain. On palpation there was
an enlarged spleen, liver and lymph nodes. A lumbar puncture showed central
nervous system involvement. The child underwent chemotherapy treatment
and is now attended by the nurse for regular routine examinations.
Which condition would require more attention from the nurse?
a- Increased leukocytes
b- Lack of muscle coordination
c- Bleeding while brushing teeth
d- Occasional nausea and vomiting

209- During the evaluation at a community clinic, the patient completes the medical
history. Which of the following is NOT a risk factor for an acute myocardial
infarction?
a- Coronary artery disease
b- Smoking
c- Hemophilia
d- Hyperlipidemia

210- A postoperative patient has the nursing diagnosis of ineffective tissue


perfusion. To assess for tissue perfusion the nurse should check all of the
following except:
a- Skin and nail bed color
b- Temperature of extremities
c- Respiratory rate
d- Peripheral pulses

55
Azyma142@hotmail.com 0560903480

211- A patient visits the clinic for a 2- week checkup after a corneal transplantation
(keratoplasty). The nurse observes the patents sclera is red and the patient
complains of the eye feeling irritated. The nurse suspects the patient may
have:
a- Infection
b- Hemorrhage
c- Graft rejection
d- Postoperative glaucoma

212- The nurse is assessing a patient who just had surgery under general
anesthesia. The patients respiration rate is 4 per minute and the O2
saturation on 3L per minute of oxygen via nasal cannula is 84%.
The nurse is awaiting the results of an arterial blood gas (ABG) and
anticipates that which of the following will be ELEVATED?
a- Arterial oxygen saturation (SaO2)
b- Hydrogen ion concentration (pH)
c- Partial pressure of arterial oxygen (PaO2)
d- Partial pressure of arterial carbon dioxide (PaCO2)

213- During the immediate postoperative period, a patient reveals an oxygen


saturation of 91%. The nurse should:
a- Position the patient on the left side
b- Administer supplemental oxygen
c- Continue to provide supportive care
d- Lower the temperature of the room

56
Azyma142@hotmail.com 0560903480

214- The nurse performs an assessment of a 23 year- old man who believes that
people are spying on him. During the interview, he keeps his eyes to the floor
and answers questions awkwardly. He has never had an intimate relationship
and avoids contact with his family members. He has never been employed and
tells the nurse that he is not looking for a job. The nurse considers Ericksons
theory of psychosocial development.
Which stage is this patient most likely experiencing?
a- Autonomy versus shame and doubt
b- Initiative versus guilt
c- Trust versus mistrust
d- Identity versus confusion

215- A 32 year- old woman with a motor neuron disease has progressive weakness
of cranial nerves V, IX, XII. She has recently been experiencing sudden
outbursts of crying and laughing. Both stroke and myasthenia gravis have
been ruled out.
Which voluntary muscle activity would be most significantly impacted?
a- Walking
b- Swallowing
c- Breathing
d- Smelling

216- Causes of primary hypothyroidism in adults include:


a- Malignant or benign thyroid nodules
b- Surgical removal or failure of the pituitary gland
c- Surgical removal or radiation of thyroid gland
d- Autoimmune-induced atrophy of the gland
57
Azyma142@hotmail.com 0560903480

217- A 72-year-old patient is diagnosed with hypothyroidism, and levothyroxine


(Synthroid) is prescribed. During initiation of thyroid replacement for the
patient, it is most important for the nurse to assess:
a- Mental status
b- Nutritional level
c- Cardiac function
d- Fluid balance

218- A client presents with Hypocalcemia, hyperphosphatemia, muscle cramps, and


positive Trousseau's sign. What diagnosis does this support?
a- Diabetes insipidus
b- Conn's syndrome
c- Hypoparathyroidism
d- Acromegaly

219- The nurse acknowledges that the first-line drug for treating of client's blood
pressure might be which drug:
a- Diuretic
b- Alpha blocker
c- ACE inhibitor
d- Alpha/beta blocker

220- The nurse is teaching a client about clopidogrel (Plavix). What is important
information to include?
a- Constipation may occur
b- Hypotension may occur
c- Bleeding may increase when taken with aspirin
d- Normal dose is 25 mg tablet per day

58
Azyma142@hotmail.com 0560903480

221- The nurse knows that the client's cholesterol level should be within which
range:
a- 150 to 200 mg/dL
b- 200 to 225 mg/dL
c- 225 to 250 mg/dL
d- Greater than 250 mg/dL

222- The nurse is performing an assessment in a client with a suspected diagnosis


of cataract. The chief clinical manifestation that the nurse would expect to
note in the early stages of cataract formation is:
a- Eye pain c- Blurred vision
b- Floating spots d- Diplopia

223- A client who has a history of Crohn's disease is admitted to the hospital with
fever, diarrhea, cramping, abdominal pain, and weight loss. The nurse should
monitor the client for:
a- Hyperalbuminemia
b- Thrombocytopenia
c- Hypokalemia
d- Hypercalcemia

224- Adult patient complains of diarrhea, vomiting, abdomen cramp and pain
within the past 2 weeks. The patient reported that the pain increases when he
eats and relieves when he passes stool. Which of the following may be the
cause:
a- Appendicitis c- Ectopic pregnant
b- Crohns disease d- Cholecystitis

59
Azyma142@hotmail.com 0560903480

225- When caring for a 3-day-old neonate who is receiving phototherapy to treat
jaundice, the nurse in charge would expect to do which of the following:
a- Turn the neonate every 6 hours
b- Encourage the mother to discontinue breast-feeding
c- Notify the physician if the skin becomes bronze in color
d- Check the vital signs every 2 to 4 hours

226- After 3 days of breast-feeding, a postpartum patient reports nipple soreness.


To relieve her discomfort, the nurse should suggest that she:
a- Apply warm compresses to her nipples just before feedings
b- Lubricate her nipples with expressed milk before feeding
c- Dry her nipples with a soft towel after feedings
d- Apply soap directly to her nipples, and then rinse

227- Which of the following interventions would be helpful to a breastfeeding


mother who is experiencing engorged breasts:
a- Applying ice
b- Applying a breast binder
c- Teaching how to express her breasts in a warm shower
d- Administering bromocriptine (Parlodel)

228- What type of milk is present in the breasts 7 to 10 days postpartum?


a- Colostrum
b- Hind milk
c- Mature milk
d- Transitional milk

60
Azyma142@hotmail.com 0560903480

229- Which of the following is a sign of rejection after a renal transplantation


operation:
a- Decrease potassium
b- Decrease sodium
c- Decrease creatinine
d- Decrease urine out put

230- Which of the following is among the signs of magnesium toxicity:


a- Cardiac arrhythmia
b- Loss of deep tendon reflex
c- Patient confusion and irritability
d- Respiratory failure

231- A nurse is caring for a client who has a potassium level of 5.4 mEq/L. The
nurse should assess the client for:
a- ECG changes
b- Constipation
c- Polyuria
d- Hypotension

232- Moderate to severe hypokalemia in a patient will cause the nurse to observe:
a- Muscle spasms and slow respirations
b- Muscle weakness and cardiac dysrhythmias
c- Confusion and irritability
d- Vomiting and diarrhea

61
Azyma142@hotmail.com 0560903480

233- Which of the following electrolytes decrease would cause PVC (premature
ventricular contraction):
a- Potassium
b- Sodium
c- Calcium
d- Magnesium

234- Which of the following electrolytes decrease would cause recurrent vomiting
and general weakness:
a- Potassium
b- Sodium
c- Calcium
d- Magnesium

235- The nurse is educating a couple who has had difficulty with conception. The
client asks about the hormone, which is responsible for the production of eggs.
The nurse answers that this hormone is called:
a- Melanocyte-stimulating hormone
b- Luteinizing hormone (LH)
c- Follicle-stimulating hormone (FSH)
d- Interstitial cell-stimulating hormone (ICSH)

236- The patient has an order to receive 45 mg of prednisone by mouth daily.


Available are 10-mg tablets. How many tablets should the nurse prepare to
give?
a- 1.6 c- 4.5
b- 2 d- 6

62
Azyma142@hotmail.com 0560903480

237- The nurse caring for a 54-year-old patient hospitalized with diabetes mellitus
would look for which of the following laboratory test results to obtain
information on the patient's past glucose control:
a- Prealbumin level
b- Urine ketone level
c- Fasting glucose level
d- Glycosylated hemoglobin level

238- Which laboratory test is most important for the nurse to monitor to determine
how effectively the clients diabetes is being managed?
a- Fasting blood glucose
b- Blood chemistry profile
c- Complete blood count
d- Glycosylated hemoglobin (HbA1c)

239- Blood sugar for new employee at company with sedentary life style and obese
and by result of Fasting blood Sugar 6.9 mmol (abnormal reading ) what is the
best next step to do to estimate his blood sugar condition:
a- Draw blood sample for HbA1c
b- Repeat the test for the next 2 days
c- Estimate the postprandial sugar level
d- Write a recommendation note for not hiring him

240- HbA1c as a measurement instrument for glucose level in the blood, it


measures blood glucose level within:
a- Last 3 months c- Last 7 months
b- Last 5 months d- Last 1 year

63
Azyma142@hotmail.com 0560903480

241- Which laboratory test indicates liver cirrhosis?


a- Decreased red blood cell count
b- Decreased serum acid phosphatase level
c- Elevated white blood cell count
d- Elevated serum aminotransferase

242- Which statement best describes the difference between the pain of angina and
the pain of myocardial infarction:
a- Pain associated with angina is relieved by rest
b- Pain associated with myocardial infarction is always more severe
c- Pain associated with angina is confined to the chest area
d- Pain associated with myocardial infarction is referred to the left arm

243- The nurse is developing a bowel-retraining plan for a client with multiple
sclerosis. Which measure is likely to be least helpful to the client?
a- Limiting fluid intake to 1000mL per day
b- Providing a high-roughage diet
c- Elevating the toilet seat for easy access
d- Establishing a regular schedule for toileting

244- Which organism is responsible for the onset of rheumatic fever?


a- Staphylococcus aureus
b- Listeriosis
c- Group A -hemolytic streptococcus
d- Epstein-Barr virus

64
Azyma142@hotmail.com 0560903480

245- For what reason are patients instructed to do Kegel exercises when they are
discharged after childbirth:
a- They help her get back in shape
b- They prevent her breasts from becoming engorged
c- They help strengthen the perineal muscles
d- They help reduce strain on the back

246- You are preparing to discharge a patient who suffered third degree burn,
which of the following instructions is appropriate to give the patient to
decrease infection:
a- You must take large amount of fluids
b- You should wash your hands thoroughly
c- You better increase vitamin intake
d- You should avoid fatty meals

247- The most effective procedure to prevent spared of infection is:


a- Wearing gloves
b- Using antiseptic
c- Sterilization
d- Hand washing

248- What is the normal pulse range for an adult?


a- 120 to 160 beats per minute
b- 90 to 140 beats per minute
c- 60 to 100 beats per minute
d- 50 to 80 beats per minute

65
Azyma142@hotmail.com 0560903480

249- A four year- old girl was playing outside in a park when she came running to
her mother crying and holding her right, upper arm. Two hours later they
presented to the emergency department. There was a swelling over the upper
right arm, with pain and itching at the site as well as swelling of the oral
mucosa. The child seems anxious.
What is the next most appropriate step in management?
a- Administer subcutaneous epinephrine
b- Maintain a patent airway
c- Administer oral diphenhydramine
d- Prepare intubation equipment

250- A 65 year- old man presents with a resting tremor in the right forearm. An
assessment of gait reveals decreased arm swinging and slight dragging of the
foot on the right side. His body movements are slow. He has not been sleeping
well at night for the past six months due to leg pain and says that he feels
constantly tired and weak. He reports that he has not suffered any recent fall
and that the symptoms seem to be slowly worsening.
Which medication is most likely to be administered?
a- Levodopa c- Phenytoin
b- Haloperidol d- Benzodiazepine

251- When planning discharge teaching for the parent of an infant with
bronchiolitis, the nurse should EMPHASIZE:
a- Use of supplemental oxygen at night
b- Frequent hand washing
c- Sleeping in the supine position
d- Rice- thickened formula during night- time feedings

66
Azyma142@hotmail.com 0560903480

252- A home care nurse visits a diabetic patient who was started on insulin
injections. Upon examination, the nurse observes small lumps and dents on
the right upper arm where the patient has injected insulin.
What is the BEST nursing intervention?
a- Refer the patient to a dermatologist for diabetic cellulitis
b- Instruct the patient to rotate the sites of injection
c- Refer the patient to an endocrinologist for better control of glucose level
d- Instruct the patient to inject in the muscular area instead of the subcutaneous
area

253- A patient has an order for 1000 milliliters (ml) of intravenous (IV) fluid to
infuse over eight hours. The available IV tubing has a drip factor of 10 gtts/ml.
Which of the following rates is correct?
a- 125 ml/hour
b- 125 drop/minute volume (ml) X drop factor (gtts / ml)
= gtts / min
---------------------------------------------
c- 21 drops/minute (flow rate)
time (min)
d- 21 ml/hour

254- When is the correct time for the nurse to administer the childs morning dose
of a combination regular and NPH insulin?
a- 30 minutes before breakfast is served
b- 15 minutes before breakfast is served
c- 30 minutes after breakfast is served
d- 15 minutes after breakfast is served

67
Azyma142@hotmail.com 0560903480

255- The client has an order for administration of 10 units of regular insulin to be
given at 7:00 a.m. The nurse should offer a snack at:
a- 3:00 p.m. c- 11:00 a.m.
b- 1:00 p.m. d- 9:00 a.m.

256- The client is being treated with NPH insulin at 8:00 a.m. The nurse should
offer a snack at:
a- 9:00 a.m. c- 6:00 p.m.
b- 11:00 a.m. d- 2:00 p.m.

257- A client with diabetes comes to the emergency department. The nurse obtains
a blood glucose measurement with a glucometer and notes that it is 510
mg/dL. The physician orders I.V. insulin. Which type of insulin can be given
both intravenously and subcutaneously?
a- Regular c- Lente
b- NPH d- 70/30

258- If a client with type 1 (insulin-dependent) diabetes mellitus receives 5 units of


NPH insulin every morning at 7 a.m., the nurse should closely monitor the
client for signs of hypoglycemia at what time:
a- 3:00 p.m. c- 7:30 a.m.
b- 12:00 a.m. d- 10:00 p.m.

259- NPH is an example of which type of insulin:


a- Short-acting
b- Intermediate-acting
c- Rapid-acting
d- Long-acting

68
Azyma142@hotmail.com 0560903480

260- A 70 years- old woman presents with increasing dyspnea on exertion. She feels
breathless and restless while performing household tasks, such as making the
bed and sweeping the floor. Her previous medical history includes a
myocardial infarction at 57 years old. She sleeps with her head elevated on
three pillows. Examination reveals bilateral basal crackles and cold, damp
skin.
Blood pressure 172/94 mmhg
Heart rate 94/min
Respiratory rate 36/min
Temperature 37.1 C
Oxygen saturation 90% on room air
Which heart chamber most likely failed first?
a- Right atrial
b- Right ventricle
c- Left ventricle
d- Left atrial

261- The nurse is preparing a discharge plan for a 65 year- old man with a new
diagnosis of congestive heart failure. The discharge orders include furosemide
40 milligrams by mouth twice per day. The nurse recommends food to reduce
unwanted medication side effects.
Which of the following food would be most appropriate to include in the
teaching plan?
a- Green and leafy vegetables
b- Bananas and oranges
c- Chicken and fish
d- Whole wheat grains

69
Azyma142@hotmail.com 0560903480

262- A child comes into the clinic with several lesions to the scalp. The round
lesions have dandruff like scaling with hair loss.
What is the MOST likely diagnosis for this skin condition?
a- Impetigo
b- Ringworm (tinea capitis)
c- Pediculosis capitis
d- Scabies

263- A 40 years- old woman suffered an automobile accident one month ago. The
accident resulted in a C-4 spinal cord injury. Her preventive care includes
independent daily performance of coughing and deep breathing.
Which range- of- motion exercise would be most beneficial for this patient?
a- Active
b- Passive
c- Combined
d- Resistive

264- A nine year- old girl has a nursing diagnosis of altered body image related to
changes in appearance secondary to varicella infection. The childs body is
covered with a rash and many large, weeping pustules. The nurse provides
counseling to the mother who is concerned that the child will develop scarring.
Which intervention is most appropriate?
a- Keep the skin out of direct sunlight
b- Apply calamine lotion to skin every two hours
c- Wash pustules with soap and keep dry
d- Soak in colloidal oatmeal bath three times daily

70
Azyma142@hotmail.com 0560903480

265- A nurse is assessing a client with possible Cushing's syndrome. In a client with
Cushing's syndrome, the nurse expects to find:
a- Weight gain in arms and legs
b- Thick, coarse skin
c- Hypotension
d- Deposits of adipose tissue in the trunk and dorsocervical area

266- Which vitamin deficiency is most likely to be a long-term consequence of a


full-thickness burn injury?
a- Vitamin A c- Vitamin C
b- Vitamin B d- Vitamin D

267- Sildenafil (Viagra) is prescribed to treat a client with erectile dysfunction. A


nurse reviews the client's medical record and would question the prescription
if which of the following is noted in the client's history:
a- Neuralgia c- Use of nitroglycerin
b- Insomnia d- Use of multivitamins

268- The physician ordered 1 liter of normal saline infusion for four hours, how
much N/S should be delivered to the patient in one hour:
a- 300 ml c- 200 ml
b- 250 ml d- 150 ml

269- Best diagnostic test for suspected leukemia is:


a- Bone marrow aspiration
b- Blood chemistry
c- CBC
d- PT AND PTT

71
Azyma142@hotmail.com 0560903480
270- The primary nursing goal when caring of a child with leukemia is to:
a- Take vital signs
b- Provide good nutrition
c- Give analgesics
d- Prevent infection

271- The oxygen flow rate for the nasal cannula is considered to be:
a- 1-6 liter/m
b- 3-8 liter/m
c- 5-10 liter/m
d- 10-15 liter/m

272- If the client develops a thrombus in one of the leg veins, which client response
would the nurse expect when eliciting Homans sign?
a- Sharp, immediate calf pain
b- Sudden numbness in the foot
c- Inability to bend the knee when asked
d- Tingling throughout the affected leg

273- When the nurse assesses the client for Homans sign, which technique is most
accurate?
a- Have the client push each foot against the mattress.
b- Have the client extend the legs and flex each foot toward the head
c- Ask the client to sit up in bed and point all the toes forward
d- Ask the client to contract the thigh muscles

274- The phase that include action of the nursing care plan is called:
a- Implementation c- Assessment
b- Diagnosis d- Evaluation
72
Azyma142@hotmail.com 0560903480

275- After vein ligation and stripping operation, the patient should:
a- Wear elastic stocking
b- Do exercises
c- Rest the leg
d- Avoid walking

276- Rabies disease is considered:


a- Viral disease
b- Bacterial disease
c- Protozoan disease
d- Fungal disease

277- A client receiving IV infusion. The skin around the IV insertion site is red,
warm to touch and painful. The nurse should first:
a- Discontinue the IV
b- Slow the IV rate for 30 minutes then reassess the site
c- Place a cold compression on the area
d- Place a warm compression on the area

278- Clients who have casts applied to an extremity must be monitored for
complications. The most significant complication for which the nurse should
assess the client's extremity is:
a- Warmth
b- Numbness
c- Skin desquamation
d- Generalized discomfort

73
Azyma142@hotmail.com 0560903480

279- A nurse is assessing the legs of a client who's 36 weeks pregnant. Which
finding should the nurse expect?
a- Absent pedal pulses
b- Bilateral dependent edema
c- Sluggish capillary refill
d- Unilateral calf enlargement

280- Which of the following is the best nursing action to assess pulse in a toddler
patient:
a- Assess the brachial artery with the middle finger and index
b- Assess the Dorsalis pedis artery with your palm
c- Assess the carotid artery with the four fingers
d- Assess the radial artery using three finger

281- One nursing intervention for patient with productive cough is to facilitate
removal of secretions. This can be done by:
a- Encourage the patient to perform slow and shallow breathing
b- Encourage the patient to increase fluid intake
c- Encourage the patient to hyperventilate
d- Encourage the patient to decrease fluid intake

282- A nurse examines a clients level of responsiveness. She finds that the patient
opens his eyes to verbal commands, obeys verbal commands, and is oriented to
time, place, and person. Whats the clients Glasgow Coma Scale:
a- 11/15
b- 12/15
c- 13/15
d- 14/15

74
Azyma142@hotmail.com 0560903480

283- A 73 year old patient looks at you when you speak to her. When you ask her
the date, she says "blue." You note left-sided weakness when she grips your
fingers. What is her Glasgow Coma Score?
a- 11/15
b- 12/15
c- 13/15
d- 14/15

284- A patient during examination opens his eyes in response to pain, makes no
verbal response, but withdraws from pain. What is the Glasgow Coma Score
(GCS) for this patient?
a- 3 c- 7
b- 5 d- 11

285- A newborn APGAR score at 1 and 5 minutes is 5 and 10, half an hour later the
baby became bluish in color with heart rate of 140/m, your first action would
be:
a- Estimate the score again
b- Shower the baby with warm water
c- Give oxygen immediately
d- Ignore the finding because it is normal

286- The primary critical observation for Apgar scoring is the:


a- Heart rate
b- Respiratory rate
c- Presence of meconium
d- Evaluation of the Moro reflex

75
Azyma142@hotmail.com 0560903480

287- When performing a newborn assessment, the nurse should measure the vital
signs in the following sequence:
a- Pulse, respirations, temperature
b- Temperature, pulse, respirations
c- Respirations, temperature, pulse
d- Respirations, pulse, temperature

288- Within 3 minutes after birth the normal heart rate of the infant may range
between:
a- 100 and 180
b- 130 and 170
c- 120 and 160
d- 100 and 130

289- The expected respiratory rate of a neonate within 3 minutes of birth may be as
high as:
a- 50
b- 60
c- 80
d- 100

290- When performing nursing care for a neonate after a birth, which intervention
has the highest nursing priority:
a- Obtain a dextrostix
b- Give the initial bath
c- Give the vitamin K injection
d- Cover the neonates head with a cap
76
Azyma142@hotmail.com 0560903480

291- Which of the following instructions are appropriate for the nurse to give the
patient about the time of taking omeprazole tablet:
a- Take the tablet during the meal
b- Take the tablet 2 hours after the meal
c- Take the tablet 30-45 minute before the meal
d- Take the tablet before bed time

292- Patient obtained second degree burn to his abdomen and his entire back,
according to rule of nine what is the percentage of burn of total body surface
area:
a- 9%
b- 18%
c- 27%
d- 36%

293- R.T.A. Patient arrived to E.R, on examination the patient has increased heart
rate, low blood pressure and decreased level of consciousness, you must think
that the patient is developing:
a- Coma
b- UTI
c- Shock
d- Respiratory arrest

294- Which of the following disease has no vaccinations:


a- Small pox
b- Measles
c- Poliomyelitis
d- Chicken pox
77
Azyma142@hotmail.com 0560903480

295- The type of burn in which all the dermis and epidermis, is destroyed and there
is involvement of underlying structures is called:
a- Superficial or first degree burn
b- Partial thickness or second degree burn
c- Full-thickness or third degree burn
d- Fourth degree burn

296- Which of the following is the drug of choice for treatment of rheumatic fever:
a- Amoxicillin c- Cephalexin
b- Azithromycin d- Penicillin

297- What disease is associated with a Vitamin C deficiency?


a- Pellegria
b- Neural tube defects
c- Scurvy
d- Pitted edema

298- A 59 year old woman is diagnosed with Hemolytic anemia. What foods would
be recommended for her to eat:
a- Fish oils c- Yellow fruits
b- Eggs, cheese d- Grains, nuts

299- The physician orders an intradermal injection of 5 tuberculin units/0.1 ml of


tuberculin purified derivative. Which needle is appropriate for this injection:
a- 12 gauge needle
b- 18 gauge needle
c- 22 gauge needle
d- 26 gauge needle

78
Azyma142@hotmail.com 0560903480

300- Which nursing diagnosis takes priority for a newly diagnosed patient with a
left- sided stroke?
a- Risk of impaired swallowing related to absent gag reflex
b- Risk for impaired skin integrity related to immobility
c- Risk for infection related to invasive line placement
d- Risk for impaired speech related to left sided stroke

301- When caring for a patient with left-sided homonymous hemianopsia resulting
from a stroke, which intervention should the nurse include in the plan of care
during the acute period of the stroke?
a- Apply an eye patch to the left eye
b- Approach the patient from the left side
c- Place objects needed for activities of daily living on the patient's right side
d- Reassure the patient that the visual deficit will resolve as the stroke progresses

302- A client with a serum glucose level of 618mg/dl is admitted to the facility. Hes
awake and oriented, has hot dry skin, and has the following vital signs:
Temperature 38.1 C
Heart rate 116 beats/minute
Blood pressure 108/70 mm Hg
Based on these assessment findings, which nursing diagnosis takes the highest
priority?
a- Deficient fluid volume related to osmotic diuresis
b- Decreased cardiac output related to elevated heart rate
c- Imbalanced nutrition: less than body requirements related to insulin deficiency
d- Ineffective thermoregulation related to dehydration

79
Azyma142@hotmail.com 0560903480

303- A patient has a blood glucose level of 60mg/dl. The patient is to receive 15 g of
carbohydrate if the blood glucose is less than 70mg/dl.
How many mls of orange juice should the patient receive?
a- 120 ml
b- 90 ml
c- 60 ml
d- 30 ml

304- A patient has a blood glucose level of 60mg/dl. The patient is to receive 15 g of
carbohydrate if the blood glucose is less than 70mg/dl.
How many candies should the patient receive?
a- 4 candies
b- 6 candies
c- 8 candies
d- 10 candies

305- A patient is scheduled for a pneumonectomy in the morning.


Which of the following diagnosis is the MOST likely indication for this type
of surgery?
a- Lung carcinoma
b- Pulmonary tuberculosis
c- Benign pulmonary nodule
d- Mediastinal shift

80
Azyma142@hotmail.com 0560903480

306- The nurse is caring for a patient with a deep vein thrombosis (DVT). The
patients heparin sodium infusion has been discontinued and the patient is
receiving prescribed warfarin sodium (Coumadin)
The nurse should advise the patient that which of the following needs to be
continued?
a- Daily complete blood count (CBC)
b- Laboratory tests for partial thromboplastin time (PTT)
c- Strict Bedrest
d- Wearing elasticized support stockings

307- A patient receiving a total parenteral nutrition through a central line suddenly
has difficulty breathing and is restless. Chest auscultation reveals a heart
murmur.
Blood pressure 90/60 mmhg
Heart rate 120/min
Respiratory rate 22/min
Temperature 37.1 C
Oxygen saturation 90% on room air
What is the most appropriate initial nursing action?
a- Notify the physician
b- Administer 100% oxygen by face mask
c- Place patient in left side lateral decubitus
d- Obtain stat blood glucose measurement

81
Azyma142@hotmail.com 0560903480

308- A 16 year- old boy fractured his right tibia and fibula during a football match.
Eight weeks later, the fractures were successfully healed and the cast was
removed.
Which range- of- motion exercises would be most beneficial?
a- Active c- Combined
b- Passive d- Resistive

309- A 65 year- old woman who was diagnosed with hypothyroidism at the age of
45 is brought to the clinic by ambulance. On arrival, she had a severely
decreased level of consciousness. Her breathing is shallow and irregular. The
skin is cool, dry and pale. There is generalized non- pitting edema of all
extremities and face.
Blood pressure 70/40 mmhg
Heart rate 60/min
Respiratory rate 12/ min
Body temperature 35.5 C tympanic
What is the most appropriate method to re-warm this patient?
a- Warmed intravenous isotonic fluids
b- Place the patient is a warm bath
c- Apply heat packs to the head and neck
d- Cover the patient with hospital blankets

310- The physician ordered a blood glucose test for the neonates the nurse knows
the best site to puncture is usually:
a- The lateral heel c- Finger tip
b- Anterior sole d- Anterior scalp

82
Azyma142@hotmail.com 0560903480

311- The nurse is teaching a patient who was recently diagnosed with rheumatoid
arthritis affecting the hands.
Which of the following treatment should the nurse discuss with the patient
at this time?
a- Transcutaneous electric nerve stimulation (TENS)
b- Iontophoresis
c- Nonsteroidal anti-inflammatory drugs (NSAIDs)
d- Synovectomy

Problems The patient will


A Altered tissue perfusion related to hypertension Maintain blood pressure below 130/90 mmhg
B Fluid volume excess related to lower extremity edema Not have signs of edema or unexpected weight gain
C Knowledge deficit related to disease process, hyper Verbalize understanding of disease process,
tension symptoms and medication management
D Alternation in comfort related to chronic headaches Report an absence of headaches each week
312- A patient has come to the clinic for follow- up one week after being discharged
from the hospital for treatment of a hypertensive crisis. Blood pressure
stabilized at 124/78 mm Hg. The patient reports feeling well, has no edema, no
longer has daily headaches. Blood pressure is 156/90 mm Hg. During
evaluation the patient admits to having stopped taking medication that had
been ordered because headaches are no longer present. Unless the symptoms
return, the patient states he will not be returning to the clinic.
What should the nurse do?
a- Resolve and discontinue the entire care plan per patient request, suggest
psychology consult
b- Resolve B and D; continue A and C
c- Continue entire care plan as written
d- Add a new problem to the care plan, non-compliance and interventions to
determine potential reasons.

83
Azyma142@hotmail.com 0560903480

313- A 34 year- old quadriplegia patient resides at home with his wife. In order to
prevent contractures of all extremities, the community care nurse will instruct
the patients wife in performance of:
a- Active range of motion exercises
b- Passive range of motion exercises
c- Active- assistive range of motion exercises
d- Resistive range of motion exercises

314- A six years- old patient has presented to the clinic with fever, malaise, and
anorexia. The patient was treated 2 weeks ago for a streptococcal infection of
the throat.
The nurse should expect the physician to order what test?
a- Electrocardiogram
b- Jones test
c- Spinal tap
d- Heart biopsy

315- A 40 year- old woman presented with right hip pain. Palpation of the pelvic
girdle is normal. An X- ray shows bone deformities, with osteolytic lesions and
bone enlargement. The patient has not suffered any trauma and has been
generally healthy.
Which serum laboratory analysis would be most useful?
a- Prothrombin time
b- Alkaline phosphatase
c- Acid phosphatase
d- Parathyroid hormone

84
Azyma142@hotmail.com 0560903480

316- A client complains of itching about 20 minutes after he began his blood
infusion, which of the following is the best nursing action his caring nurse
must take:
a- Stop the infusion immediately and call the physician
b- Call the physician immediately and the stop the infusion
c- Culminate the client and continue infusion
d- Continue the infusion while monitoring the client closely

317- A 32 year- old woman with diabetes mellitus type 1 underwent a


cholecystectomy and is now on day two of recovery. The patients bowel
sounds have returned and she has resumed a normal diet but has been
finishing less than half of each meal on the tray. The nurse enters the room to
perform a routine assessment and finds the patient confused and shaky
Blood pressure 110/60 mmhg
Heart rate 96/min
Respiratory rate 22/min
Temperature 37.0 C oral
What is the most appropriate initial intervention?
a- Administer glucagon
b- Notify the physician
c- Give an insulin injection
d- Offer a glass of juice

318- A Cancer patient wants to stop chemotherapy and change to palliative care,
this situation indicates which psychological status:
a- Depression c- Anxiety
b- Hopelessness d- Restlessness

85
Azyma142@hotmail.com 0560903480

319- What do we call the type of treatment of terminal illness patients?


a- Conservative
b- Curative
c- Palliative
d- Selective

320- Psychiatric patient appear violent for himself and others was put in the room
alone during the period of exacerbation, then patient calm down and informed
the nurse I am ok now let me with others, but the nurse refused that as a
punishment way. At which underline Label will nurse accused under the
court:
a- Abandonment
b- False imprisonment
c- Negligence
d- Duty to act

321- Which of the following actions is the first priority of care for a client
exhibiting signs and symptoms of coronary artery disease?
a- Decrease anxiety
b- Enhance myocardial oxygenation
c- Administer sublingual nitroglycerin
d- Educate the client about his symptoms

322- What is the first intervention for a client experiencing MI?


a- Administer morphine
b- Administer oxygen
c- Administer sublingual nitroglycerin
d- Obtain an ECG
86
Azyma142@hotmail.com 0560903480

323- A patient complains of stable angina presented with chest pain. The physician
ordered Nitroglycerine tablet sublingual. The nurse knows that the right dose
to give the patient is:
a- 0.1 mg c- 0.4 mg
b- 0.2 mg d- 0.8 mg

324- Cancer patient during the shift and he surrounded by visitor and laughing
loudly before the shift end patient call the nurse and asked for pain
medication and said that his level of pain 8/10 (sever pain) patient appears not
on pain, what is the best intervention for this situation:
a- Give the patient extra dose of his pain medication
b- Ask the patient about the last time he had his medication
c- Ignore the patient as you are going to leave
d- Check the patients file for history of drug abuse

325- A patient with pain was prescribed Ibuprofen 300 mg q 4 hours PRN and
morphine 5mg q 4 hours PRN. After 2 hours of receiving Ibuprofen the
patient still complaining of pain, what is the best intervention:
a- Wait 2 hours and give a second dose of ibuprofen
b- Give the patient his dose of morphine
c- Wait 1 hour and give the patient his morphine
d- Ask the patient to bear the pain

326- Which instruction should a nurse give the parents of an infant undergoing
cleft lip repair?
a- Offer the pacifier as needed
b- Lay the infant on his abdomen for sleep
c- Sit the infant up for each feeding
d- Loosen arm restraints every hour

87
Azyma142@hotmail.com 0560903480

327- Patient with abdominal incision, seven days postoperatively the patient has
pain at the site of incision which increased by walking and the incision appear
red and theres pus surrounding the incision site. This may indicate which of
the following:
a- Normal finding
b- Inflammation of the incision
c- Under nutrition
d- Good indicator

328- A preschooler is admitted to the hospital the day before scheduled surgery.
This is the childs first hospitalization. Which action will best help reduce the
childs anxiety about the upcoming surgery:
a- Begin preoperative teaching immediately
b- Describe preoperative and postoperative procedures in detail
c- Give the child dolls and medical equipment to play out the experience
d- Explain that the child will be put to sleep during surgery and wont feel anything

329- Which teaching aid provided by the nurse is developmentally appropriate for
a preschooler who is about to have a bone marrow puncture:
a- Dolls or puppets
b- Pamphlets or booklets
c- Colored diagrams
d- Commercial videotapes
330- A nurse is caring for an infant with spina bifida. Which assessment findings
suggest hydrocephalus?
a- Depressed fontanels and suture lines
b- Deep-set eyes, which appear to look upward only
c- Rapid increase in head size and irritability
d- Motor and sensory dysfunction in the foot and leg
88
Azyma142@hotmail.com 0560903480

331- Parents of a newborn with a unilateral cleft lip are concerned about having
the defect repaired. The nurse explains that a child with a cleft lip usually
undergoes surgical repair:
a- Immediately after birth
b- By 3 months of age
c- After 12 months of age
d- Varies in every case

332- The hepatitis B vaccine series should begin at what age:


a- Newborn c- 6 months
b- 2 months d- 12 months

333- A school age client admitted to the hospital because of decreased urine output
and per-orbital edema and diagnosed with glomerulonephritis. Which of the
following interventions would receive the highest priority:
a- Monitoring the vital signs every 4 hours
b- Monitoring intake and output every 12 hours
c- Monitoring the client weight daily
d- Monitoring serum electrolyte daily

334- The nurse is caring for a client with renal failure who is being treated with
peritoneal dialysis. Which assessment before and after peritoneal dialysis is
most valuable in evaluating the outcome of treatment:
a- Pulse rate
b- Body weight
c- Abdominal girth
d- Urine output
89
Azyma142@hotmail.com 0560903480

335- Which finding provides the best evidence that peritoneal dialysis is achieving a
therapeutic effect:
a- Urine output increases
b- Appetite improves
c- Potassium level falls
d- Red blood cell count is lower

336- When the nurse is advising the client about the potential complications
associated with peritoneal dialysis, which complication is most important to
include:
a- Pulmonary edema
b- Abdominal peritonitis
c- Abdominal hernia
d- Ruptured aorta

337- A client receiving hemodialysis treatment arrives at the hospital. He


complains of shortness of breath, and pedal edema is noted. His last
hemodialysis treatment was yesterday.
Blood pressure 200/100
Heart rate 110/min
Respiratory rate 36/m
Oxygen saturation 89% on room air
Which of the following interventions should be done first?
a- Administer oxygen
b- Elevate the foot of the bed
c- Restrict the clients fluids
d- Prepare the client for hemodialysis
90
Azyma142@hotmail.com 0560903480

338- Changes in personality and judgment are often associated with a lesion in
which of the following:
a- Frontal lobe
b- Parietal lobe
c- Broca's area
d- Wernicke's area

339- Anterior lobe of the brain is responsible for which of the following:
a- Personality
b- Movement
c- Speech
d- Memory

340- A history of smoking, abnormal permanent enlargement of the alveoli, cough,


and dyspnea suggest:
a- Asthma
b- Emphysema
c- Chronic bronchitis
d- Obstructive sleep apnea

341- Which of the following is the end of the first stage of labor:
a- Cervix dilated to 10 cm
b- Crowning of the presenting part
c- Increased bloody show
d- Contractions lasting up to 60 seconds

91
Azyma142@hotmail.com 0560903480

342- During the latent phase of the first stage of labor, how often should the nurse
plan to assess the fetal heart rate?
a- Every 5 to 15 minutes
b- Every 15 to 30 minutes
c- Every 30 to 60 minutes
d- Every 60 to 90 minutes

343- The nurse explains that oxytocin (Pitocin) is given after delivery of the baby
and placenta for which purpose:
a- To increase the blood pressure
b- To prevent the uterus from inverting
c- To decrease the likelihood of hemorrhage
d- To prevent rupture of the uterus

344- A nurse in the delivery room is assisting with the delivery of a newborn infant.
After the delivery of the newborn, the nurse assists in delivering the placenta.
Which observation would indicate that the placenta has separated from the
uterine wall and is ready for delivery:
a- The umbilical cord shortens in length and changes in color
b- A soft and boggy uterus
c- Maternal complaints of severe uterine cramping
d- A sudden gush of dark blood from the introitus

345- Which vitamin deficiency increase the risk of neural tube defects:
a- Folic acid
b- Vitamin C
c- Vitamin B12
d- Calcium

92
Azyma142@hotmail.com 0560903480

346- A client has been diagnosed with folic acid deficiency. The nurses discharge
teaching will focus on foods high in folic acid. Which of the following foods has
the highest folic acid level:
a- Citrus fruits
b- Raisins
c- Brewers yeast
d- Eggs

347- Which of the following is the recommended daily dose of folic acid during
pregnancy:
a- 200 mcg
b- 400 mcg
c- 800 mcg
d- 1000 mcg

348- Lochia normally disappears after how many days postpartum:


a- 5 days
b- 7-10 days
c- 18-21 days
d- 28-30 days

349- If the maintenance requirement of fluid for a child is 900 ml/ day, each daily
feeding account 120 ml, what is the number of feeding is required to achieve
this maintenance:
a- 5 feedings
b- 6 feedings
c- 7 feedings
d- 8 feedings
93
Azyma142@hotmail.com 0560903480

350- A 12 year- old patient had a cast removed from the left leg after wearing if for
eight weeks. The patient wants to resume sports as soon as possible. In order
to regain muscle strength lost while wearing cast, the nurse will instruct the
patient in performance of:
a- Resistive range of motion exercises to left leg
b- Passive range of motion exercises to right leg
c- Active- assistive range of motion exercises to the right leg
d- Active range of motion exercises to both legs

351- A 31 years- old woman with diabetes type 1 presents to the clinic with fatigue,
blurred vision, and loss of appetite. Her breath smells like fruit and she leaves
the room twice during the examination to use the toilet. She has brought a
little bottle of water with her that she finishes while at the clinic. She reports
that she has had a cold for the past three days, but has not taken additional
insulin during the illness
Blood pressure 130/70 mmhg
Heart rate 90/min
Respiratory rate 20/min
Body temperature 38.0 C oral
What is the most appropriate nursing diagnosis
a- Risk for impaired skin integrity related to circulation
b- Deficient knowledge related to illness management
c- Risk for fluid volume excess related to fluid intake
d- Imbalanced nutrition related to decreased appetite

94
Azyma142@hotmail.com 0560903480

352- Adult patient suffers from hoarseness of voice, and difficulty in speech may be
suffering from an injury of which of the following cranial nerves:
a- IX c- XI
b- X d- XII

353- The parent of a child with chronic asthma is hesitant to discipline because the
child often doesnt feel well. The nurse should encourage the parent to:
a- Set consistent behavior limits
b- Be more lenient during times of illness
c- Cherish the limited time the child has to live
d- Avoid upsetting the child with limit- setting

354- A 55 year- old woman presented with pain in the joints of the hands. She feels
generally tired and has had a low grade fever for the past week. On
examination, the joints of the fingers on both hands are symmetrically swollen.
Blood pressure 120/70 mmhg
Heart rate 82/min
Respiratory rate 18/min
Temperature 38.0 C
Oxygen saturation 99% on room air
Which type of arthritis is most likely?
a- Rheumatoid
b- Septic
c- Psoriatic
d- Osteoarthritis

95
Azyma142@hotmail.com 0560903480

355- A patient presented to the office for a physical examination. The patient is
found to be healthy and fit but occasionally drinks alcohol and has
unprotected sex.
What is the BEST nursing diagnosis?
a- Health- seeking behavior
b- Knowledge deficit, high- risk behaviors
c- Low self- esteem
d- Altered thought process

356- A seven year- old child presented to the emergency room with a fracture of the
right arm.
What would be the long- term goal for this patient?
a- Patient verbalizes decreased pain level
b- Nurse notes neurovascular checks are within normal limits
c- Nurse notes decreased swelling of the right arm
d- Patient performs activities of daily living without difficulty

357- The nurse is caring for a woman whose husband beats her regularly. Which is
the most important long-term goal for this woman?
a- Provide a long-term support group
b- Help her feel like a survivor
c- Point out the ways she behaved
d- Be able to blame the abuser

358- In what position should a dyspneic patient be placed?


a- Prone
b- Recumbent
c- Semi-fowlers
d- Trendelenburg
96
Azyma142@hotmail.com 0560903480

359- A nurse is admitting a two year- old child with an umbilical hernia.
Which of the following interventions does NOT meet the childs
developmental needs?
a- Allowing the child to make choices when possible
b- Providing rooming in the unlimited visitation
c- Attempting to continue rituals used at home
d- Maintaining strict bedrest

360- A six year- old boy is in the postoperative care unit following a tonsillectomy.
The nurse observes that his gag reflex has returned and removes the artificial
airway. The patient then begins to cry and tells the nurse that his throat hurts
badly. What type of data is the patient providing the nurse?
a- Objective c- Comparative
b- Inferential d- Subjective

361- A first time mother of a three week- old breastfed baby brings the infant to the
clinic and complains that her child has been forcefully vomiting after feeding.
He was born 40 weeks gestation. Weighing 3.6 kilograms. He is constantly
hungry and irritable. Examination reveals a swollen abdomen and a palpable
mass in the middle upper right quadrant.
What is the most likely health problem?
a- Intussusception
b- Pyloric stenosis
c- Gastroeosophageal reflux
d- Diaphragmatic hernia

97
Azyma142@hotmail.com 0560903480

362- A home care nurse reviews the second follow- up laboratory test results of a
patient with iron deficiency anemia. The nurse compares the recent laboratory
results to the results taken 3 months ago. The patient has been eating food rich
in iron as prescribed by the nutritionist.
What can be inferred about the progress of the treatment by comparing the
two laboratory values?
Laboratory values three months prior
Test Result Normal value
Albumin 35 34-56 g/L
Glucose 5.3 3.5-6.5 mmol/L
HCT 0.32 0.41-0.50
Hb 121 120-140 g/L
Blood urea nitrogen 6.1 2.8 to 8.9 mmol/L
Creatinine 60 58-145 mol/L
Current laboratory values
Test Result Normal value
Albumin 33 34-56 g/L
Glucose 5.7 3.5-6.5 mmol/L
HCT 0.41 0.41-0.50
Hb 139 120-140 g/L
Blood urea nitrogen 5.7 2.8 to 8.9 mmol/L
Creatinine 60 58-145 mol/L
a- Patients uptake of iron by diet is sufficient, and no additional intervention is
necessary
b- Patients uptake of iron by diet is sufficient, but additional intervention is
necessary
c- Patients uptake of iron by diet is insufficient, and additional intervention is
necessary
d- Patients uptake of iron by diet is insufficient, and the physicians immediate
action is necessary

98
Azyma142@hotmail.com 0560903480

363- Which assessment finding would the nurse expect in an infant diagnosed with
pyloric stenosis?
a- Abdominal rigidity
b- Ribbon-like stools
c- Visible waves of peristalsis
d- Rectal prolapse

364- A six year-old child with type 1 diabetes has an uncontrolled blood glucose
level. The child has been given regular insulin with minimal change in glucose
level in the first 30 minutes.
Which type of insulin has an onset of 15 minutes and a duration of no more
than 4 hours?
Type of insulin Onset(minutes) Maximal Duration
a- Aspart (Novolog) activity(hours) (hours)

b- Lispro (Humalog) Aspart(Novolog) 15 1-3 3-5


Lipsro(Humalog) 10-30 1-2 2-4
c- Semilete regular 30-60 2-4 6-9
semilente 30-60 2-4 10-12
d- Neutral protamine hagedom (NPH) NPH 120 4-12 24
Lente 120 8-10 24

365- A patient with cerebrovascular accident, left-sided hemiplegia, and aphasia


has nursing diagnosis of risk for aspiration related to swallowing difficulties
with a short-term goal that the patient will not aspirate. The patient has
undergone insertion of a percutaneous gastric endoscopy and has all nutrition
and fluid administered through the tube. The nurse should:
a- Continue the care plan as written
b- Discontinue the risk for aspiration diagnosis
c- Revise the goals and interventions for the diagnosis
d- Add a nursing diagnosis of ineffective health maintenance

99
Azyma142@hotmail.com 0560903480

366- A nurse is evaluating a patient 5 days after a right total hip replacement.
Which of the following goals is appropriate for the patient?
a- Maintain abduction without dislocation
b- Rest with legs elevated while sitting
c- Tie shoes and put on undergarments without assistive devices
d- Perform scissor-like exercises daily

367- A three year- old child was admitted to the postoperative care unit following a
heart transplant. The nurse administers cyclosporine by intravenous infusion.
Fifteen minutes later the child has difficulty breathing, his skin feels cold and
clammy and he appears restless. Which is the most appropriate initial nursing
action?
a- Ensure airway patency
b- Administer oxygen therapy
c- Discontinue intravenous infusion
d- Administer intramuscular epinephrine

368- A 40 year-old woman is a gravida2, para 2 and is currently trying to conceive.


Her previous pregnancy resulted in the birth of a baby with cleft lip and
palate. The patient is anxious and concerned about future pregnancies and the
nurse provides genetic counseling and reassurance.
Which food would most effectively prevent recurrence?
a- Green vegetables and citrus fruit
b- Eggs, milk and dairy products
c- Wheat, corn, rice, oats and rye
d- Beef, chicken and yellow vegetables

100
Azyma142@hotmail.com 0560903480

369- A 50 year-old woman presented with poor balance and coordination. She says
that she has developed pain on the outer aspect of the thighs and the inner side
of the arch of the foot. The nurse wishes to examine the integrity of the
affected lumbar spinal nerve root and performs a deep tendon reflex
examination.
Which reflexes would most likely be diminished?
a- Planter
b- Brachioradialis
c- Patellar
d- Achilles tendon

370- When administrating an intramuscular injection to an infant, which of the


following sites is appropriate for the nurse to use?
a- Rectus femoris c- Dorsogluteal
b- Deltoid d- Ventrogluteal

371- A 66 years-old woman is admitted to the hospital with a history of


hypertension. She presented with breathing difficulties that worsen with
activity and while sleeping. She is generally weak and feels that her heart
misses beats and that it sometimes beats loudly. An electrocardiogram shows
atrial fibrillation, right ventricular hypertrophy and deviation toward the
right.
Which nursing intervention is most appropriate for this patient?
a- Provide a bedside commode (portable toilet)
b- Place in right side-lying position
c- Encourage family and friends to visit
d- Encourage independent hygienic activities

101
Azyma142@hotmail.com 0560903480

372- Oral iron supplements are prescribed for a 6-year-old child with iron
deficiency anemia. The nurse instructs the mother to administer the iron with
which of the following food item to enhance absorption of iron:
a- Milk
b- Water
c- Apple juice
d- Orange juice

373- 9 months old infant was brought to the hospital by his parents because he is
crying most of the time. The examination revealed that he is suffering from
otitis media. Which of the following instructions you would include in the care
plan you suggest to the parents:
a- Give antibiotic as prescribed
b- Try to calm down your baby
c- Wash the ears regularly
d- Ignore the childs pain

374- A child with pneumonia was prescribed penicillin injection. Before giving the
injection to the child the nurse performed skin allergy test which showed that
the child has penicillin allergy. Which of the following is the next step the
nurse should take:
a- Stop the treatment and inform the doctor
b- Administer penicillin intramuscular
c- Postpone the does until the child is well
d- Ignore the test result and give the injection

102
Azyma142@hotmail.com 0560903480

375- The following table represents the blood pressure value for a patient in 3
successive days. Regarding the information given in the tablet, this patient is
considered: First day 120/80
a- Normal blood pressure Second day 122/87
b- Pre-hypertension
Third day 133/88
c- Hypertensive
d- Hypotensive

376- A patient with pneumonia with excessive mucous in the left lung, which of the
following is the best position to facilitate drainage of the mucous from the
lower left lung:
a- On the right side with trendelenburg position
b- On the left side with elevation of HOB
c- On the right side with elevation of the HOB
d- On the left side with trendelenburg position

377- A patient complains of left eye redness and itching, the doctor told you to put
atropine eye drops for the patient to examine his eye. The nurse should instill
the eye drops into:
a- The left eye
b- The right eye
c- Both right and left eyes
d- Neither of the eyes

103
Azyma142@hotmail.com 0560903480

378- The district nurse visits a 30 year-old woman at home following the delivery of
her second child, a full term girl. Following the delivery of her first child, she
had developed a breast infection and stopped breastfeeding because of the
pain. She asks the nurse how she can best prevent it with this infant.
What is the most appropriate response?
a- Provide feeding on demand
b- Apply vitamin E cream to the nipples
c- Request a prophylactic antibiotic
d- Apply heat to the breasts after feeding

379- In planning home care for an immunocompromised child, the nurse instructs
the parents to use cream or emollients to prevent or manage dry and cracked
skin. A parent will BEST demonstrate understanding of the rationale for this
be stating:
a- Creams will prevent breaks in the skin and decrease the chance of infection
b- Pleasantly scented creams will mask other less pleasant smells
c- Micronutrients in the creams will help prevent malnutrition
d- Creams will help prevent dehydration when my child does not drink enough

380- The nurse is teaching a patient about spironolactone (Aldactone).


Which of the following instructions should review with the patient?
a- Increasing the intake of foods that are high in potassium
b- Taking the medication right before going to sleep
c- Avoid seasonings that are labeled as salt substitutes
d- Scheduling the medication so that a multivitamin is taken an hour later

104
Azyma142@hotmail.com 0560903480

381- A 49 year-old man with a diagnosis of alcoholic cirrhosis and ascites is


discharged from the hospital with a priority nursing diagnosis of altered
nutrition, less than body requirements.
Which dietary plan is most appropriate?
a- Low-protein, low sodium
b- Low sodium, high- protein
c- High- protein, low potassium
d- Low potassium, high protein

382- A patient who is 4 days postoperatively after a total hip replacement surgery,
is obese and has not been able to ambulate since the surgery. The patient is
now diaphoretic, has chills, and complains of pain in the thigh. The MOST
likely cause is:
a- Wound infection
b- Deep vein thrombosis (DVT)
c- Pulmonary edema
d- Dehydration

383- Which statement by the patient with hyperlipidemia shows a basic


understanding of the disease and its treatment?
a- Exercises has no effect on cholesterol levels
b- Hyperlipidemia is usually asymptomatic until significant target organ damage is
done
c- HDL cholesterol level of greater than 60mg/dL increases the chance of coronary
artery disease
d- Cholestyramine (questran) should be taken in the morning with other
medications

105
Azyma142@hotmail.com 0560903480

384- Which of the following describe the correct sequence of nursing process:
a- Assessment, planning, diagnosis, implementation and evaluation
b- Planning, assessment, diagnosis, implementation and evaluation
c- Planning, diagnosis, implementation, assessment and evaluation
d- Assessment, implementation, evaluation, planning and diagnosis

385- A 14 year-old boy presented to the emergency department. He complains of


having progressively worsening stomach pain for the past eight hours. On
assessment, the pain is localized in the lower right quadrant. He rates it as a
level ten on the one-ten pain scale. Abdominal palpation shows rebound
tenderness in the lower right quadrant, and positive McBurney and psoas
signs.
Blood pressure 134-78 mmhg
Heart rate 88/min
Respiratory rate 24/min
Temperature 38.2 C
Test result normal values
WBC 12.4 4-10.5 x 109
Which intervention would be most appropriate to alleviate this patients
pain?
a- Apply warm packs to affected area
b- Maintain semi-fowlers with legs up
c- Withhold solids and liquids
d- Administer analgesics

106
Azyma142@hotmail.com 0560903480

386- Female patient with a diagnosis of atrial fibrillation, the physician ordered
you to give her digoxin IV, but the patient refused to be injected and told that
she feel she is going to vomit, what you should do:
a- Change the IV digoxin to oral form
b- Force the patient to accept the medication
c- Inform the doctor about the situation
d- Dont give any medications to this patient

387- 40 years old client complaining from abdominal pain, laboratory result for
stool analysis showed that theres occult blood in the stool. Which of the
following procedure would help for diagnosis of this patient:
a- Abdominal ultrasound c- Colonoscopy
b- Abdominal x ray d- MRI

388- A 7-year-old client is brought to the E.R. Hes tachypneic and afebrile and has
a respiratory rate of 36 breaths/minute and a nonproductive cough. He
recently had a cold. From his history, the client may have which of the
following:
a- Acute asthma
b- Bronchial pneumonia
c- Chronic obstructive pulmonary disease (COPD)
d- Emphysema

389- Which of the following is a priority goal for the client with COPD:
a- Maintaining functional ability
b- Minimizing chest pain
c- Increasing carbon dioxide levels in the blood
d- Treating infectious agents
107
Azyma142@hotmail.com 0560903480

390- A 34-year-old woman with a history of asthma is admitted to the emergency


department. The nurse notes that the client is dyspneic, with a respiratory rate
of 35 breaths/minute, nasal flaring, and use of accessory muscles. Auscultation
of the lung fields reveals greatly diminished breath sounds. Based on these
findings, what action should the nurse take to initiate care of the client:
a- Initiate oxygen therapy and reassess the client in 10 minutes.
b- Draw blood for an ABG analysis and send the client for a chest x-ray.
c- Encourage the client to relax and breathe slowly through the mouth
d- Administer bronchodilators

391- Which of the following is the best breathing pattern you should teach a client
with COPD:
a- Pursed-lip breathing c- Abdominal breathing
b- Deep breathing d- Slow lite breathing

392- A nurse instructs a female client to use the pursed-lip method of breathing and
the client asks the nurse about the purpose of this type of breathing. The nurse
responds, knowing that the primary purpose of pursed-lip breathing is to:
a- Promote oxygen intake
b- Strengthen the diaphragm
c- Strengthen the intercostal muscles
d- Promote carbon dioxide elimination

393- An infant with tetralogy of Fallot is experiencing an attack involving cyanosis


and dyspnea. Which position should the infant be placed in:
a- Fowler's c- Trendelenburg's
b- Knee-chest d- Prone

108
Azyma142@hotmail.com 0560903480

394- A client is unresponsive and has been brought to the emergency department.
Initial laboratory results reveal:
Serum K+ 3.6 mmol/L
Glucose 26 mg/dl
Hemoglobin 12. 6 gm/dl
Carbon dioxide 26. 2 mmol/L. The nurse will anticipate:
a- Dextrose by mouth
b- 50% dextrose IV
c- 1 unit of packed cells
d- 10 mEq of KCl over 0.5 hour

395- When a client has a nephrostomy tube, the priority nursing care is to:
a- Ensure drainage of urine
b- Milk the tube every 2 hours
c- Keep an accurate record of intake and output
d- Instill 2 ml of normal saline solution every shift

396- Which of the following is the most common symptom of myocardial infarction
(MI):
a- Chest pain c- Edema
b- Dyspnea d- Palpitations

397- A client has surgery for a perforated appendix with localized peritonitis. In
which position should the nurse place the client:
a- Sims position
b- Trendelenburg
c- Semi-fowlers
d- Dorsal recumbent

109
Azyma142@hotmail.com 0560903480

398- Which of the following position should the client with appendicitis assume to
relieve pain:
a- Prone
b- Sitting
c- Supine
d- Lying with legs drawn up

399- A women breast feed her infant for one or two hours and her infant cries most
of the time and she feels pain in her breast, which of the following instructions
are appropriate for the nurse to give the mother:
a- Regulate breast feeding every 3 hours
b- Thats normal feeding problem
c- Shift to bottle feeding
d- Start weaning your baby

400- While caring for a child with a ventriculoperitoneal shunt revision, the nurse
finds the patient lying with the head and feet flexed back. The nurse should
call for help and prepare for a (n):
a- Spinal tap
b- Shunt culture
c- Electrocardiogram
d- Ventricular tap

401- While assessing a child with pulmonary stenosis, the nurse should give
PRIORITY to:
a- Deep tendon reflexes
b- Urinary output
c- Exercise tolerance
d- Pattern of food intake
110
Azyma142@hotmail.com 0560903480

402- A 25 years-old male patient suffered a spinal cord injury at the T-4 level and is
being cared for in hospital. The nurse enters the patients room and finds the
patient sitting upright and looking anxious and restless. He complains of
sudden headache and nausea. Sweat forms on his forehead yet his feet are cool
to touch
Blood pressure 150/100 mmhg
Heart rate 55/min
Respiratory rate 28/min
Temperature 37.1C
What nursing intervention is initially most appropriate?
a- Assess for a full bladder
b- Lower the head of the bed 30 degrees
c- Loosen clothing and bed sheets
d- Apply heating pad to lower extremities

403- An 80 year-old man presents to the hospital with chronic fatigue, dyspepsia
and constipation. On examination he is jaundiced, has red palms, dilated veins
around the umbilicus, the abdomen is very distended and he has black, tarry
stool on a rectal exam. He is noted to be lethargic and have a flat tone. A
paracentesis reveals clear colored fluid with low protein content.
What is most likely diagnosis?
a- Gilberts syndrome
b- Thalassemia
c- Sickle cell crisis
d- Cirrhosis

111
Azyma142@hotmail.com 0560903480
404-
Admission Report at 22:45 (10:45p.m.) Diagnosis
14 years old admitted Remains febrile, maximum temperature Bacterial
18 hours ago this shift 39.3C pneumonia
Respiration rate 12-14 per minute
Heart rate 100-180 per minute
Blood pressure level within baseline
normal range
Next dose of intravenous anti-infective is
due at 08:00 (8:00 a.m.)
Refer to the accompanying figure.
The nurse should plan to monitor which of the following at least every 4
hours?
a- Vital signs c- Food intake
b- Urinary output d- Level of activity

405- Which instruction take priority in reducing anxiety related to surgery?


a- Surgical procedure and postoperative exercises
b- Risk of infection after surgery
c- Advanced directive and what it means
d- Pre-operative laboratory result and what to expect on it

406- When caring for a patient with an ostomy, the nurse knows that extra
protection for peristomal skin is MOST important for those patients with a
(an):
a- Ileostomy
b- Ascending colostomy
c- Transverse colostomy
d- Sigmoid colostomy

112
Azyma142@hotmail.com 0560903480

407- Which of the following nursing diagnosis takes PRIORITY for a patient
admitted to the critical care unit with diabetic ketoacidosis?
a- Deficient fluid volume secondary to hyperglycemia
b- Risk of infection secondary to weakened immune system
c- Deficient knowledge of cause and prevention
d- Imbalanced nutrition related to hyperglycemic state

408- A 69 year-old man is admitted to the intensive care unit following cardiac
surgery. Two hours after admission, the nurse performs a routine assessment
and notes the patients chest tube drainage is 200 milliliters and a dark red
color. He has had 60 milliliters output from the indwelling urinary catheter
Blood pressure 138/68 mmhg
Heart rate 76/min
Respiratory rate 16/min
Body temperature 37.0C oral
Oxygen saturation 94% 6L/min nasal cannula
Which finding should be reported to the doctor?
a- Dark red chest tube drainage
b- Urinary output
c- Oxygen saturation
d- Chest tube output volume

409- To minimize a toddler from scratching and picking at healing skin graft, the
nurse should utilize:
a- Mild sedatives c- Punishment for picking
b- Hand mittens d- Distractions

113
Azyma142@hotmail.com 0560903480

410- The following pain medications are ordered for a patient who had a right leg
amputation: Oxycodone 5 mg every 4 hours as needed and morphine 5 mg
every 4 hours as needed. The nurse administered oxycodone 2 hours ago, but
the patient reports pain rated 8 on a scale of 0 (no pain) to 10 (severe pain) as
the dressing change begins
After evaluating the effectiveness of the pain medication, what action should
the nurse take?
Blood pressure 169/98 mmhg
Heart rate 112/min
Respiratory rate 22/min
Temperature 36.7C
a- Administer additional oxycodone 5 mg
b- Administer morphine 5 mg
c- Change the dressing quickly
d- Ask the patient to wait 2 hours

411- A 55 year-old man presented to the clinic with complaints of numbness and
tingling in his feet for the past 3 months. He appears thin and his skin is pale.
Examination confirms the loss of vibration sense. He also has altered
proprioception. He had a subtotal gastrectomy two years ago (see lab results)
Test result normal values
MCH 2.72 1.45-2.01 fmol/cell
HCT 0.31 0.41-0.50
MCV 105 81-97 fl
Reticulocyte count 1.5 0.5%-2.5%
Which supplement would most likely improve his condition?
a- Iron c- Folate
b- B12 d- Calcium
114
Azyma142@hotmail.com 0560903480

412- A 35 year-old man is hospitalized following a blunt chest injury. He has chest
pain, breathing difficulty and asymmetrical lung expansion. Chest
auscultation reveals decreased lung sounds on the left side. The jugular veins
are distended and there is tracheal deviation to the right.
Blood pressure 92/54 mmhg
Heart rate 120/min
Respiratory rate 24/min
Temperature 36.2C
SaO2 88% on oxygen
Which initial intervention is most appropriate?
a- Send patient for X-ray examination
b- Administer bronchodilators
c- Administer intravenous fluid bolus
d- Prepare for needle decompression

413- A patient is being followed in the clinic for hypertension, adult onset diabetes,
and obesity. The patient is apathetic about learning about nutritional
guidelines to reach the goals of weight loss and consumption of a healthy diet.
The patient admitted to eating whatever is put in front of me
Which of the following actions would the nurse take?
a- Collaborate with the patient to set goals
b- Add a nursing diagnosis of non-compliance
c- Refer for psychiatric screening for depression
d- Discuss nutritional interventions with the spouse

115
Azyma142@hotmail.com 0560903480

414- The nurse is measuring the chest tube drainage of a patient who had open
heart surgery 4 hours ago.
Which of the following is the MAXIMUM hourly amount of chest tube
drainage is expected in this timeframe?
a- 100 ml
b- 200 ml
c- 300 ml
d- 400 ml

415- A nurse is caring for a three year-old child with a fractured arm.
Which of the following interventions is the MOST appropriate for pain
management?
a- Administer analgesics when necessary
b- Assess pain once a shift
c- Anticipate pain and intervene early
d- Encourage the use of self-quieting techniques

416- After a hearing restoration operation, a patient has no signs of complications


and soon recovers.
Which of the following is an expected outcome 5 days after the hearing
restoration surgery?
a- Regain full hearing
b- Minimal facial nerve paralysis
c- Minimal urinary incontinence
d- Ambulates without difficulty

116
Azyma142@hotmail.com 0560903480
417-
Goals- for the next three months, the patient will:
A Not have any migraine headaches
B Decrease the number of migraine headache
C Learn to tolerate her migraine headaches
D Continue to take prescribed preventative medications daily
E Initiate complementary pain management strategies as needed
F Wean self from all pain medications
G Keep a pain diary and recognize triggers and auras before migraine headaches commence
An adolescent patient with a history of migraine headaches for the past
year has been followed for pain management. The headaches have
decreased in severity and now occur only occasionally
What are the BEST three long term goals?
a- A, D, F
b- B, D, F
c- C, E, F
d- D, E, G

418- A community health nurse visits a patient who had a cerebrovascular


accident. The patient is at risk for deficient fluid volume due to voluntary
reduction of fluid intake to avoid the use of the bathroom. The nurse educates
the patient on the importance of drinking fluids and maintaining hydration/
Which of the following indicates the efficacy of the nursing intervention?
a- Amber color urine
b- Respiration of 35
c- Tachycardia
d- Moist mucous membrane

419- A patient with diabetes mellitus and multiple sclerosis has been prescribed
baclofen (Lioresal). The nurse knows this medication may result in?
a- Increased insulin needs c- Optic neuritis
b- Renal failure d- Muscle tremors
117
Azyma142@hotmail.com 0560903480
420-
Nursing diagnosis
A Impaired physical mobility
B Activity intolerance
C Alternation of comfort
D Risk of alternation in skin integrity
E Knowledge deficit
F Decreased cardiac output
A patient with Alzheimers disease and severe cardiomyopathy presents to the
hospital with a fractured left hip. The patient is on bed rest until a cardiologist
clears the patient to have surgery.
Which nursing diagnosis have the highest priority?
a- E and F
b- D and C
c- C and F
d- D and F

421- A 19 year-old woman telephones the nurse and complains of difficulty of


breathing. The symptoms developed while exercising that morning. She has a
productive cough with thick mucus secretions and wheezing. The nurse hears
that the breathing problems do not interfere with talking on the phone. The
woman plans to attend the clinic but must wait for two hours before she can
be driven there.
Which treatment should be recommended first by the physician?
a- Purse-lip breathing
b- Deep breathing and coughing
c- Decrease physical activity
d- B2 agonist administration

118
Azyma142@hotmail.com 0560903480

422- A patient is recovering from surgery using spinal anesthesia. The patient
developed a spinal headache.
Which of the following nursing actions would be MOST appropriate?
a- Elevate the head of the bed 30 degrees
b- Keep the patient well hydrated
c- Limit intake of salty foods
d- Lower the temperature of the room

423- A patient is preparing for a total knee replacement. During the preoperative
interview process the patient reports an allergic reaction to penicillin.
Which of the following is considered a side effect and not a true allergy to
medication?
a- Shortness of breath
b- Tingling lips and tongue
c- Rash
d- Upset stomach

424- Prior to providing care for a hospitalized infant, the nurse who focuses on
preventive measures must:
a- Introduce self to parents
b- Perform hand hygiene
c- Have a witness present
d- Assess the childs developmental level

425- As identified by Dr. Elizabeth Kubler-Ross, which stage of dying is


characterized by the transition from, NO, not me to Yes, me, but
a- Anger c- Acceptance
b- Depression d- Bargaining

119
Azyma142@hotmail.com 0560903480

426- A 50 year-old male presents to the medical office 3 weeks after cardiac surgery
with complaints of a feeling of weakness, difficulty breathing, and joint pains.
Upon examination the nurse finds a fever and a friction rub on auscultation of
the chest. The nurse recognizes that the MOST likely surgical complication is:
a- Neuropsychological dysfunction
b- Postpericardiotomy syndrome
c- Cardiac tamponade
d- Phrenic nerve damage

427- A patient with heart failure has the following vital signs, which of these vital
signs should be reported to the physician prior to administrating the next dose
of digoxin?
Blood pressure 136/84 mmhg
Heart rate 48/min
Respiratory rate 20/min
Temperature 37.1C
a- Blood pressure
b- Pulse
c- Temperature
d- Respiratory rate

428- The floating ribs that are not attached with sternum are:
a- Ribs 7&8
b- Ribs 9&10
c- Ribs 11&12
d- Ribs 9&10&11&12

120
Azyma142@hotmail.com 0560903480

429- A pediatric nurse is providing medication instructions to the mother of a 13


year-old boy who is starting anti-depressant therapy. The mother appears
unfocused, agitated and confused and asks the nurse to repeat the instructions
several times. She tells the nurse that she is concerned that she might
administer the medication incorrectly. The nurse is concerned that the mother
is not following the instructions.
Which intervention would be most appropriate?
a- Reassure her that doubts are normal
b- Reinforce the importance of correct dosage
c- Refocus the teaching with printed material
d- Take a small break and the continue

430- A child is treated for superficial (first-degree) thermal burns to the thigh. The
child is in great discomfort and does not eat.
Which of the following diagnosis should receive PRIORITY?
a- Altered nutrition
b- Impaired skin integrity
c- Risk of infection
d- Acute pain

431- A 3-week-old infant is hospitalized with jaundice. When considering the fluid
needs relative to body size of the infant as compared to the fluid needs of an
adult. The nurse knows this infant requires:
a- Less fluids
b- More fluids
c- Same amount of fluids
d- Much less fluids

121
Azyma142@hotmail.com 0560903480

432- Which of the following statement is most accurate?

a- Girls have more ADHD than boys


b- Boys are more frequently diagnosed with chronic illnesses
c- Boys are less frequently diagnosed with chronic illnesses
d- Girls have more other chronic illness than ADHD

433- What is the percentage of boy with attention deficit hyperactivity disorder?

a- 12%
b- 16%
c- 8%
d- 4%

122
Azyma142@hotmail.com 0560903480

434- The nurse is observing a detailed neurological assessment on a client with a


suspected brain tumor. When performing the Romberg test, the client sways
when the eyes are both open and closed. What does this indicate?
a- The problem is probably in the cerebellum
b- It is a position sense abnormality
c- This is not an abnormal test result
d- The client has lost proprioception

435- An 18 year-old woman who broke her right ankle is seen in the physicians
office one week after the cast was removed.
Which of the following is the short term goal for this patient?
a- Walk 100 feet with crutches
b- Walk completely independent
c- Relieve the pain
d- Strict bedrest

436- A client with multiple sclerosis has been prescribed the drug baclofen
(Lioresal). What is the action of this drug?
a- Reduces spasticity
b- Skeletal muscle relaxation
c- Immune suppression
d- Prevents viral infections

437- What is the average incubation period of Hepatitis A?


a- 30 days
b- 60 days
c- 50 days
d- 14 days
123
Azyma142@hotmail.com 0560903480

438- After the health care team meets to discuss the clients nursing needs, the
nursing diagnosis Disturbed body image is added to the care plan. The best
rationale for adding this nursing diagnosis to the care plan in the case of a
female is that females with Cushings syndrome typically experience which
physiologic effect?
a- Masculine characteristics
b- Heavy menstrual flow
c- Extreme weight loss
d- Large, pendulous breasts

439- A 64-year-old client with uterine cancer is scheduled to undergo an abdominal


hysterectomy under general anesthesia. Before the client returns from the
postanesthesia care unit, the registered nurse asks the licensed practical nurse
to help revise the care plan for the client who has undergone a hysterectomy.
Which nursing diagnosis is most appropriate for the nurse to add to the
clients care plan at this time?
a- Risk for ineffective airway clearance
b- Risk for imbalanced nutrition
c- Ineffective coping
d- Impaired verbal communication

440- The nurse initiates a teaching plan for the client with Parkinsons disease.
Which instruction should be the nurses priority in this situation?
a- Steps to enhance the clients immune system
b- Importance of maintaining a balanced diet
c- Need to remove all safety hazards
d- Importance of social interactions

124
Azyma142@hotmail.com 0560903480

441- The charge nurse enters the nursing diagnosis Risk for ineffective airway
clearance related to an inability to swallow on the clients care plan. Which
nursing intervention is most appropriate for managing the identified
problem?
a- Keeping the client supine
b- Removing all head pillows
c- Performing oral suctioning
d- Providing frequent oral hygiene

442- The client has returned from surgery with a leg cast, and the nurse is assisting
the client back to bed. Which of the following would the nurse identify as the
highest priority when documenting the postoperative circulation status of the
recently casted extremity?
a- Adequate neurovascular functioning
b- Minimal pain on movement
c- Vital signs within normal limits
d- No drainage noted on the case

443- After X-rays are taken of the head, neck, and spine, the client is diagnosed
with a head injury and admitted for inpatient care. When assessing the client
with a head injury, which of the following should receive priority attention?
a- Lung sounds
b- Clarity of speech
c- Mobility of fingers
d- Pupillary responses

125
Azyma142@hotmail.com 0560903480

444- The client experiencing a severe allergic reaction becomes pulseless. The nurse
shakes the client, shout the clients name but gets no response, and activates
the emergency medical response system. Which nursing action becomes the
next priority?
a- Administer a single blow to the sternum
b- Give two quick breaths that make the chest visibly rise
c- Begin chest compressions at a rate of 100 per minute
d- Administer an epinephrine (Adrenalin) injection

445- A 50-year-old client is scheduled for a heart transplant tomorrow. The night
nurse is asked to review the surgical procedure with the client. Because of the
clients anxiety, the client has difficulty comprehending the nurses
information. During the postoperative period, what is the best rationale for
the nurse frequently assessing the clients fluid status?
a- Urine retention is common after a heart transplant
b- Urine output is an indication of perfusion to the kidneys
c- Hydration determines when the client needs to be transfused
d- Hydration indicates when fluids should be increased

446- A nurse is assigned to care for a patient with a diagnosis of thrombotic stroke.
The nurse knows that this type of stroke is MOST LIKELY caused by:
a- Blockage of large vessels as a result of atherosclerosis
b- Emboli produced from valvular heart disease
c- Decreased cerebral blood flow due to circulatory failure
d- A temporary disruption in oxygenation of the brain

126
Azyma142@hotmail.com 0560903480

447- A nurse obtains a urine dipstick analysis sample from a 35 year-old woman
who reports having burning sensation with urination and a sense of urgency
and frequency. She had been diagnosed with the condition six months
previously and was prescribed a course of antibiotics.
Urinalysis Results Normal values
Colour Dark yellow Straw-coloured
Odour Abnormal Almost nothing
Appearance Turbid Clear
Leukocyte esterase Positive Negative
Nitrites Positive Negative
Which type of pharmacological treatment is most likely?
a- Anti-viral
b- Anti-fungal
c- Anti-bacterial
d- Anti-parasitic

448- A 30 year-old woman has been prescribe albuterol PRN and prophylactic
inhaled corticosteroids to be taken once per day. She has been taking the
prophylactic as prescribed but has needed to use albuterol more often than
usually. She has a chronic cough and often has air hunger.
Which intervention is initially most appropriate?
a- Refer for a chest X-ray
b- Administer magnesium sulfate
c- Assess peak flow measure
d- Perform arterial blood gas sampling

449- Which of the following drugs is not used in the treatment of pulmonary
embolism?
a- Heparin c- Digoxin
b- Warfarin d- Streptokinase

127
Azyma142@hotmail.com 0560903480

450- A home care patient with chronic obstructive pulmonary disease (COPD)
reports an upset stomach. The patient is taking theophylline (Theo-Dur) and
triamcinolone acetonide (Azmacort). The nurse should instruct the patient to
take:
a- Theo-Dur on an empty stomach
b- Theo-Dur and Azmacort at same time
c- Theo-Dur and Azmacort 12 hours apart
d- Theo-Dur with milk or crackers

451- A 65 year-old woman presented to her care provider with complaints of bright
red blood in the stool, a loss of appetite, a feeling of fullness and fatigue. She
had lost 5 kilograms in the past three weeks without dieting. A faecal occult
blood test is positive and the patient is scheduled for additional screening test.
Which screening test is most likely for this patient?
a- Barium enema
b- Colonoscopy
c- Endoscopy
d- Computed tomography scan

452- A baby girl was born prematurely at 33 weeks gestation due to placenta
abruption. She is now two months old and has a potential hearing deficit. In
the neonatal intensive care unit (NICU) she had passed the initial screening
tests for hearing but has significant risk factors that suggest further
evaluation.
Which autonomic reflex should be further evaluated?
a- Moro c- Pupillary
b- Babinski d- Tonic neck

128
Azyma142@hotmail.com 0560903480

453- A 45 year-old woman presented with a generalized rash that is not itchy. She
reports that she has had the problem for the past 15 years. Examination
reveals a well-outlined, reddish plaque over the right gluteal fold. The plaque
has scales over it and is cracked in some areas.
Which intervention is initially appropriate?
a- Apply topical cream to the affected area
b- Expose area to sunlight for twenty minutes daily
c- Maintain immunosuppressant therapy regimen
d- Increase dietary intake of vitamin A

454- The patient is receiving mechanical ventilation set at fraction of inspired


oxygen (FiO2) 100%.
The nurse should understand that which of the following can improve this
patients oxygenation?
a- Adding positive end expiratory pressure (PEEP)
b- Place the patient in trendelenburg position
c- Increasing the FiO2
d- Suctioning the patient hourly

455- A nurse is caring for a 3-week-old infant who just admitted to the hospital.
Which of the following nursing interventions does NOT support this infants
basic emotional and social needs?
a- Provide for continual contact between parents and infant
b- Actively involve parents in caring for the infant
c- Keep the infants environment quite, dim and free of sensory stimulation
d- Foster infant-sibling relationships as appropriate

129
Azyma142@hotmail.com 0560903480

456- A community health nurse is teaching a health class about infectious diseases
process.
The nurse instructs the class that rabies would be considered which of the
following type of infection?
a- Viral
b- Protozoan
c- Fungal
d- Bacterial

457- A couple who wants to conceive but has been unsuccessful during the last 2
years has undergone many diagnostic procedures. When discussing the
situation with the nurse, one partner states, We know several friends in our
age group and all of them have their own child already, why cant we have
one? Which of the following would be the most pertinent nursing diagnosis
for this couple?
a- Fear related to the unknown
b- Pain related to numerous procedures
c- Ineffective family coping related to infertility
d- Self-esteem disturbance related to infertility

458- A 64 year old male client with a long history of cardiovascular problem
including hypertension and angina is to be scheduled for cardiac
catheterization. During pre-cardiac catheterization teaching, the nurse should
inform the client that the primary purpose of the procedure is:
a- To determine the existence of CHD
b- To visualize the disease process in the coronary arteries
c- To obtain the heart chambers pressure
d- To measure oxygen content of different heart chambers

130
Azyma142@hotmail.com 0560903480

459- A gravida 3 para 2 is admitted to the labor unit. Vaginal exam reveals that the
client's cervix is 4cm dilated. The patient complains of pain which she stated
to be 7/10.at this time and until a full cervix dilatation is achieved, what is the
priority nursing goal for such patient at this time is:
a- Pain management
b- Prevent fetal distress
c- Preparing the patient for anesthesia
d- Keeping this patient NPO

460- A client with iron deficiency anemia is scheduled for discharge. Which
instruction about prescribed ferrous gluconate therapy should the nurse
include in the teaching plan?
a- Take the medication with an antacid.
b- Take the medication with a glass of milk.
c- Take the medication with cereal.
d- Take the medication on an empty stomach.

461- After 4 hours of active labor, the nurse notes that the contractions of a
primigravida client are not strong enough to dilate the cervix. Which of the
following would the nurse anticipate doing?
a- Obtaining an order to begin IV oxytocin infusion
b- Administering a light sedative to allow the patient to rest for several hour
c- Preparing for a cesarean section for failure to progress
d- Increasing the encouragement to the patient when pushing begins

131
Azyma142@hotmail.com 0560903480
462- A 24 year-old woman was prescribed loratidine (Claritin) 10 mg tablet q 12
hours a.c. for allergy.
The nurse reviews the medication order and explains to the patient the
relation of this drug to meals is as following:
a- The drug is to be taken after meals
b- The drug is to be taken during meals
c- The drug is to be taken before meals
d- The drug is to be taken away from meals

463- An elderly client is experiencing an alteration in his equilibrium and


coordinated muscle movements. The nurse realizes that these functions are
controlled by which area of the nervous system?
a- Brain stem
b- Cerebrum
c- Diencephalon
d- Cerebellum

464- A 32-year-old female is admitted for a hemorrhoidectomy. During the nursing


assessment, all of the following factors are elicited. Which one is most likely to
have contributed to the development of hemorrhoids?
a- The client states that she usually cleans herself from back to front after a bowel
movement
b- The client says her mother and grandmother had hemorrhoids
c- The client has had four pregnancies
d- The client eats bran every day

132
Azyma142@hotmail.com 0560903480

465- A client who has hepatitis A asks, How could I have gotten this disease?
what is the nurses best response?
a- You may have eaten contaminated food
b- You could have contracted the disease by using intravenous drugs
c- You must have received and infected blood
d- You could have contracted the disease by engaging in unprotected sex

466- A client undergoes right mastectomy for carcinoma. When teaching the client
post-mastectomy exercises, it is important for the nurse to:
a- Exercise both arms simultaneously
b- Exercise the right arm only
c- Have the client wear a sling between exercise periods
d- Wait until the incision has healed

467- The following picture represents a newborn reflex known as?


a- Rooting reflex
b- Moro reflex
c- Grasping reflex
d- Startle reflex

468- The nurse is caring for a client who has had a right modified radical
mastectomy this morning. Which exercise should the nurse encourage the
client to perform this evening?
a- Hair combing exercises with the right arm
b- Wall climbing exercises with the right arm
c- Movement of the fingers and wrists of the right arm
d- Exercises of the left arm only

133
Azyma142@hotmail.com 0560903480

469- A 38-year-old client who has mitral stenosis is hospitalized for a valve
replacement. Which condition is the client most likely to report having had
earlier in life?
a- Meningitis
b- Syphilis
c- Rheumatic fever
d- Rubella

470- An 82-year-old woman who has Alzheimers disease is admitted to the acute
care unit. She frequently gets out of bed and wanders in the hall, unable to
find her way back to her room. She even gets in the beds of other clients. What
nursing action is most appropriate for this client?
a- Restrain her so she will not wander in the halls
b- Ask her roommate to call the nurse whenever she leaves the room
c- Punish her when she gets in a bed other than her own
d- Put her favorite picture on the door to her room

471- A 35-year-old man is admitted with severe renal colic. The nurse should
monitor this man for possible complications. Which of the following is a
complication of renal colic?
a- Anemia
b- Polyuria
c- Hypertension
d- Oliguria

134
Azyma142@hotmail.com 0560903480

472- A nurse discusses high-risk complications with a group of women at a prenatal


clinic. Which client would the nurse identify as being at highest risk for
developing complications during pregnancy?
a- A 25-year-old gravida I client
b- A client with the placenta implanted on the fundus of the uterus
c- A client who has nausea and vomiting during the first trimester
d- A 30-year-old client with DM

473- A 56 year-old man was brought to the emergency room by his relatives, on
examination he appears sick and has severe weakness. One of his relatives told
the nurse that the man eats nearly nothing and have been crying most of the
time for 3 months now since he lost his son in an accident. The nurse knows
that this patient is suffering from:
a- Mania
b- Insomnia
c- Depression
d- Schizophrenia

474- Which of the following drugs is used for the treatment of digitalis toxicity?
a- Protamine sulfate
b- Streptokinase
c- Theophylline
d- Digoxin immune Fab

135
Azyma142@hotmail.com 0560903480

475- A female client is admitted with a diagnosis of acute renal failure. She is
awake, alert, oriented, and complaining of severe back pain, nausea and
vomiting and abdominal cramps.
Blood pressure 100/70 mm Hg
Pulse 110
Respiratory rate 30
Temperature 38C oral
Sodium 120 mEq/L
Potassium 5.2mEq/L
Urinary output for the first 8 hours is 50 ml
The client is displaying signs of which electrolyte imbalance?
a- Hyponatremia
b- Hyperkalemia
c- Hyperphosphatemia
d- Hypercalcemia

136
Azyma142@hotmail.com 0560903480
Some important notes to remember
Renal failure acute: low protein, high carbohydrate, low sodium (oliguric phase),
high protein, high calorie, restricted fluid (diuretic phase)

Renal failure chronic: low protein, low sodium, low potassium

Normal color of stoma (colonostomy): pink or red

Teething starts at: 7 months and till 16 months

After Laparoscopic cholecystectomy: patient regain normal activity after 12-14 days

After renal surgery the most common complications: DVT

Typhoid: transmission (contact) isolation (contact isolation)

Food rich in vitamin C: orange, Broccoli, strawberries, tomato

MRSA isolation: contact

Complication of hemorrhoidectomy: infection and thrombosis

HCV transmitted by: blood

Test to assess anemia patient: ferritin and hemoglobin

Psoriasis takes the shape of: scales

137
Azyma142@hotmail.com 0560903480

Normal capillary refill: less than 3 seconds

Child has limb swelling and ascites, the main cause of swelling: Increased
hydrostatic pressure

Sign of malnutrition for child: open fontanel, low weight for his age

138

Você também pode gostar